MBE Practice Test

Few hours of practice make all the difference.

Start Free practice

iPREP - Ace That Test!

Welcome to the world of legal qualification, future attorney! If you’re reading this, you’re likely ready to embark on your journey to take the Multistate Bar Exam (MBE), a crucial stepping stone to practicing law in the United States. Let’s give you a quick overview of what you’re stepping into.

The MBE is a standardized, multiple-choice examination developed and provided by the National Conference of Bar Examiners (NCBE). It’s one component of the Uniform Bar Examination (UBE) and is used by most jurisdictions in the United States to evaluate your ability to apply fundamental legal principles and reasoning to analyze given fact patterns. In other words, the MBE is not just about rote learning of legal concepts, but also about understanding and applying them effectively.

“The MBE is 50% of the exam. Typically, when people fail the bar exam, it’s because they didn’t do well on the MBE. Therefore, much of your time and effort should go towards doing well on the MBE.”

Source: bp000000/reddit

The exam spans one full day and consists of 200 multiple-choice questions, which are split into two sections of 100 questions each. It covers seven key areas of law, namely: Civil Procedure, Constitutional Law, Contracts, Criminal Law and Procedure, Evidence, Real Property, and Torts.

While the sheer volume of questions may seem intimidating, bear in mind that the MBE is designed to be tough – it’s a test that ensures you’re equipped with the necessary knowledge and skills to navigate the complex legal landscape.

A fundamental piece of information you should remember is that the MBE is a highly valid and reliable assessment tool. According to the NCBE, the test is continuously reviewed and updated by teams of legal experts to ensure that it reflects the most current legal standards and principles. So, rest assured, the MBE isn’t a hurdle set by chance, but a purposeful evaluation of your readiness to practice law.

So, buckle up and get ready. The journey to conquering the MBE is challenging, but with the right approach and preparation, it is entirely achievable. “The MBE felt like climbing a mountain, but reaching the peak was worth every grueling step,” As a past test-taker shared on a forum. As you embark on your journey, keep in mind that this is not just a test—it’s a milestone in your legal career.

Did you know?

Each of the seven sections in the MBE comprises approximately 25 questions, evenly contributing to your score. Interestingly, the MBE is graded by a scaled scoring system where raw scores are converted to a scale ranging from 0 to 200 – with the national average typically falling around 135 to 140.

The MBE is administered twice a year, in February and July, and is taken in a single day under proctored conditions. What’s more, MBE scores are frequently used not only by state bar examiners to determine competence, but also by some employers and law schools for evaluation purposes.

MBE Question Types Explained

The Multistate Bar Exam (MBE) covers seven major areas of law, each with its unique focus and assessment objectives. Here is a breakdown of each:

1. Civil Procedure: This section assesses your understanding of jurisdiction and venue, pretrial procedures, jury trials, motions, verdicts and judgments, and appeal procedures. A firm grasp of these procedures, the ability to analyze case facts, and application of the rules of civil procedure to those facts are essential skills.

2. Constitutional Law: This section evaluates your understanding of the nature of the U.S. Constitution, its branches, and the constitutional limitations placed upon them. Success in this section requires an in-depth knowledge of the Constitution and the ability to apply constitutional principles to complex legal scenarios.

3. Contracts: This section measures your understanding of contract formation, defenses to enforceability, contract terms, performance, breach, and remedies. Critical thinking and interpretation skills are key to navigating hypothetical contractual disputes.

4. Criminal Law and Procedure: This area assesses your knowledge of criminal offenses, defenses, constitutional protection for the accused, and legal procedures in the prosecution of crimes. The ability to think like a prosecutor or defense attorney, analyzing the elements of a crime and potential defenses, is crucial here.

5. Evidence: This section evaluates your understanding of presentation of evidence, relevancy, privileges, witnesses, hearsay, and hearsay exceptions. An acute ability to discern relevant from irrelevant information, and a detailed understanding of rules governing admissibility of evidence, are necessary skills for this section.

6. Real Property: Here, you’ll be assessed on your knowledge of property rights and interests, transactions and contracts, mortgages, titles, and land use controls. A deep understanding of property law terminology and principles, along with the ability to apply these to various real property scenarios, will serve you well.

7. Torts: This section measures understanding of intentional torts, negligence, strict liability, and other torts. The ability to identify potential civil wrongs and their remedies in hypothetical situations is key in this section.

TopicNumber of QuestionsWhat It Tests
Civil Procedure~25Understanding of jurisdiction, venue, pretrial procedures, jury trials, motions, verdicts, appeals.
Constitutional Law~25Understanding of the U.S. Constitution, its branches, and their constitutional limitations.
Contracts~25Understanding of contract formation, enforceability, terms, performance, breach, and remedies.
Criminal Law & Procedure~25Understanding of criminal offenses, defenses, constitutional protection for the accused.
Evidence~25Understanding of evidence presentation, relevancy, privileges, witnesses, hearsay.
Real Property~25Understanding of property rights, transactions, mortgages, titles, and land use controls.
Torts~25Understanding of intentional torts, negligence, strict liability, and other torts.
Overview of the MBE Topics. Source: iPREP

To perform well across all these topics, you’ll need not only strong factual knowledge but also critical reading, analytical thinking, and problem-solving skills. As one test taker on a social forum noted,

“Conquering the MBE isn’t just about memorizing the law—it’s about understanding it, and being able to apply it to any curveballs the exam throws at you.”

Civil Procedure Questions

The Civil Procedure section of the MBE focuses on the rules and principles that govern the litigation process in U.S. federal courts. This section typically features around 25 questions, though the exact number may vary slightly from exam to exam.

Civil Procedure questions are designed to assess your understanding of the “nuts and bolts” of the judicial system. They delve into a range of topics including jurisdiction and venue, pretrial procedures, resolution of claims and disputes without trial, the law applied by federal courts, jury trials, verdicts and judgments, and appeal procedures.

Here’s a closer look at what these topics entail:

  1. Jurisdiction and Venue: Questions in this area assess your understanding of where a case can be heard. This includes concepts such as personal jurisdiction, subject-matter jurisdiction, venue, and the standards for removal and transfer between courts.
  2. Pretrial Procedures: This includes your understanding of procedures such as pleadings, joinder of parties and of claims, discovery processes, and summary judgment motions.
  3. Resolution of Claims and Disputes Without Trial: This area covers your understanding of processes such as settlements, alternative dispute resolution (ADR) mechanisms, and default judgments.
  4. The Law Applied by Federal Courts: Here, the questions measure your understanding of the application of federal common law, choice of law, the Erie Doctrine, and federal-state interplay.
  5. Jury Trials: This tests your knowledge of the right to a jury trial, jury selection and instructions, and the verdict process.
  6. Verdicts and Judgments: Questions in this area assess your understanding of judgment as a matter of law, renewing judgment as a matter of law, verdict forms, and the execution and relief from judgments.
  7. Appeal Procedures: This area tests your understanding of the appellate jurisdiction, final judgment rules, and standards of review.

In these questions, you might be asked to determine the proper venue for a case, decide if a judgment can be appealed, or assess whether certain evidence can be discovered. The objective is to test your ability to apply these principles to fact patterns that mirror those found in actual legal practice.

Mastering the Civil Procedure section requires a comprehensive understanding of these topics and strong analytical skills to apply this knowledge to the nuanced situations presented in the questions.

Civil Procedure Sample Question

A woman filed a lawsuit in federal court against a corporation for breach of contract, seeking damages of $100,000. The corporation is incorporated in Delaware, with its principal place of business in New York. The woman is a resident of Texas, and the alleged breach occurred in Texas. The corporation believes the case should be heard in the state court in Texas.

Which of the following is the most appropriate course of action for the corporation?

  1. Move for a change of venue to Texas federal court, since the alleged breach occurred there.
  2. Move to dismiss for lack of personal jurisdiction, since the corporation doesn’t primarily operate in Texas.
  3. Move to dismiss for lack of subject matter jurisdiction, since a federal question isn’t present.
  4. Move to remand the case to Texas state court, since that’s where the alleged breach occurred.

The correct answer is A.

The corporation can move for a change of venue to Texas federal court under 28 U.S.C. § 1404(a) if it can show that the transfer serves the interests of justice and is more convenient for parties and witnesses. Since the alleged breach occurred in Texas, the Texas federal court is a more appropriate venue.

Here’s why the other options are incorrect:

B is incorrect because a corporation is deemed to be a citizen of the state in which it is incorporated and the state where it has its principal place of business for diversity jurisdiction purposes. So, a federal court would have personal jurisdiction over the corporation.

C is incorrect because federal courts have subject-matter jurisdiction over this case. The case involves citizens of different states (the woman is from Texas and the corporation is from Delaware/New York), and the amount in controversy exceeds $75,000. Therefore, it falls within the diversity jurisdiction of the federal courts.

D is incorrect because a defendant cannot move to remand a case to state court. The term “remand” applies when a case is transferred from federal court back to state court. However, since this case was initially filed in federal court, there’s no state court action to remand to.

Constitutional Law Questions

The Constitutional Law section of the MBE is designed to assess your understanding of the fundamental principles and concepts enshrined in the United States Constitution. Similar to other sections, it typically consists of around 25 questions, although the exact number might fluctuate slightly between different test administrations.

The Constitutional Law questions fall into two broad categories: individual rights and the structure of the government. The individual rights questions focus on the constitutional protections of civil rights and liberties, while the structure of the government questions examine the relationships between different branches of the federal government and the relationships between the federal government and the states.

Here’s a closer look at the main topics covered in this section:

  1. Nature of Judicial Review: This assesses your understanding of the principles that underlie the power of judicial review, including justiciability, the power of the courts to interpret the Constitution, and the Supreme Court’s interaction with lower courts and other branches of government.
  2. Separation of Powers: This evaluates your knowledge of the distribution of powers among the legislative, executive, and judicial branches of the federal government, including checks and balances.
  3. Relations of Nation and States in a Federal System: Here, you’ll be tested on your understanding of the powers of the federal government vis-à-vis states, including the Supremacy and Commerce Clauses, and the Tenth Amendment.
  4. Individual Rights: Questions in this area assess your understanding of individual freedoms and protections provided by the Constitution, including freedom of speech, religion, equal protection, due process, and takings.

Each of these categories includes an array of subtopics. For example, under individual rights, you might face questions about the First Amendment’s guarantees of religious freedom and free speech, the Fourth Amendment’s protections against unreasonable searches and seizures, or the Fifth and Fourteenth Amendments’ due process and equal protection clauses.

These questions often involve hypothetical scenarios that require you to apply the principles of constitutional law to analyze a legal issue or resolve a legal problem. For example, you might be asked to determine the constitutionality of a law, to assess whether a government action violates an individual’s constitutional rights, or to evaluate the legality of interactions between different branches of government.

To do well on these questions, it’s important to have a thorough understanding of the U.S. Constitution, the precedents set by the Supreme Court, and the principles of constitutional interpretation.

Constitutional Law Sample Question

A city enacted a statute that bans all political protests on city streets. A group of citizens who wanted to protest city policies on a city street challenged the statute.

What is the most likely outcome?

  1. The statute will be upheld, as it is a legitimate exercise of the city’s police power.
  2. The statute will be upheld, as city streets are not a traditional public forum.
  3. The statute will be struck down, as it infringes on the citizens’ First Amendment rights.
  4. The statute will be struck down, as the city did not follow due process in enacting it.

The correct answer is C.

The First Amendment to the United States Constitution protects the right to freedom of speech, including the right to engage in political protests. While the city can impose reasonable restrictions on the time, place, and manner of protests, a total ban, especially in a traditional public forum like city streets, is likely to be seen as a violation of the First Amendment.

Here’s why the other options are incorrect:

A is incorrect because, although a city does have police power to enact laws for the welfare of its citizens, this power is limited by the Constitution, including the First Amendment. The city cannot use its police power to infringe upon protected constitutional rights.

B is incorrect because city streets are considered a traditional public forum, a place that has been devoted to assembly and debate. The rights of the state to limit speech in such a forum are sharply restricted.

D is incorrect because there’s no information given that the city didn’t follow due process in enacting the statute. However, even if due process was followed, a law that infringes on constitutional rights can still be struck down.

Contracts Questions

The Contracts section of the MBE covers fundamental principles related to contract law, and typically comprises about 25 questions, although this number may vary slightly from exam to exam.

This section primarily focuses on two areas: formation and performance of contracts, and breaches of contract and the resulting remedies. Questions are designed to assess your understanding of the creation, interpretation, and enforcement of these agreements, as well as the legal implications of a breach of contract.

Here’s a more detailed look at these topics:

  1. Formation of Contracts: This area assesses your understanding of the basic elements required for contract formation, including offer, acceptance, and consideration. It also tests your knowledge of other key aspects of contract formation, such as the intention to create legal relations, capacity to contract, legality of the agreement, and the statute of frauds.
  2. Performance, Breach, and Discharge: This evaluates your knowledge of the obligations that arise from a contract, the conditions under which these obligations must be performed, and the events that may lead to the discharge of these obligations. It also tests your understanding of the concept of breach of contract and the factors that may excuse a breach.
  3. Contract Terms and Interpretation: This tests your understanding of the different types of terms in a contract (e.g., express terms, implied terms, conditions, warranties), and how these terms are interpreted by the courts.
  4. Remedies for Breach of Contract: This area assesses your knowledge of the remedies available when a contract is breached. This includes compensatory damages, specific performance, rescission, and restitution.

Questions in this section often involve hypothetical scenarios in which you are asked to apply principles of contract law to resolve a legal issue or determine the legal consequences of a hypothetical situation. For example, you may be asked to determine whether a contract exists, whether a contract has been breached, or what remedies are available following a breach of contract.

To do well in this section, it’s crucial to have a thorough understanding of the principles of contract law, including both common law and the Uniform Commercial Code (UCC), as they apply to sales of goods contracts.

Contracts Sample Question

A homeowner entered into a contract with a contractor to remodel her kitchen for $20,000. The contract stipulated that the work would be completed in two months. Four months have passed, and the kitchen is still not complete. The homeowner has found another contractor who will finish the job for $8,000. She wishes to terminate the contract with the original contractor.

What is the most likely outcome?

  1. The homeowner can terminate the contract, but will owe the original contractor the full $20,000.
  2. The homeowner can terminate the contract and won’t owe the original contractor anything.
  3. The homeowner can terminate the contract, but will owe the original contractor for the work already done.
  4. The homeowner cannot terminate the contract as she is obligated to allow the original contractor to finish the job.

The correct answer is C.

Here, the original contractor has breached the contract by failing to complete the job within the agreed time. The homeowner is thus within her rights to terminate the contract. However, under the principle of quantum meruit, the contractor is entitled to be paid for the work he has already done.

Here’s why the other options are incorrect:

A is incorrect because, though the homeowner agreed to pay $20,000, the contractor breached the contract, so he is not entitled to the full contract price.

B is incorrect because, while the homeowner can terminate the contract, the original contractor is entitled to compensation for the work already done under the principle of quantum meruit.

D is incorrect because the contractor’s breach (i.e., not finishing the job within the agreed time) allows the homeowner to terminate the contract. The homeowner is not obligated to let the original contractor finish the job.

Criminal Law & Procedure Questions

The Criminal Law and Procedure section of the MBE primarily focuses on the fundamental principles and concepts associated with criminal law, the rules and regulations that define the nature of and punishments for offences against society. As with other sections, this part typically consists of approximately 25 questions.

The Criminal Law and Procedure questions fall into two broad categories: substantive criminal law and criminal procedure. The substantive criminal law questions focus on the principles of criminal liability, while the criminal procedure questions examine the constitutional protections afforded to criminal defendants.

Here’s a closer look at the main topics covered in this section:

  1. Substantive Criminal Law: This assesses your understanding of crimes, parties to crimes, and the various types of defenses that can be raised in a criminal case. Crimes typically tested include homicide, other crimes against the person (e.g., assault, false imprisonment, kidnapping), crimes against property (e.g., larceny, robbery, burglary, arson), and inchoate crimes (e.g., attempt, solicitation, conspiracy).
  2. Criminal Procedure: This evaluates your knowledge of the procedural safeguards that the Constitution provides to criminal defendants, such as Fourth Amendment protections against unreasonable searches and seizures, Fifth Amendment protections against self-incrimination and the right to due process, Sixth Amendment rights to counsel, to a speedy trial, to an impartial jury, to confront witnesses, and to compulsory process, and Eighth Amendment protections against cruel and unusual punishment.

Each of these categories includes an array of subtopics. For example, under criminal procedure, you might face questions about the admissibility of evidence obtained through a search, the circumstances under which a defendant can waive his rights, or the procedural requirements for a criminal trial.

These questions often involve hypothetical scenarios that require you to apply the principles of criminal law and procedure to analyze a legal issue or resolve a legal problem. For example, you might be asked to determine whether a crime has been committed, to assess whether a defense is available, or to evaluate the legality of a police action.

To do well on these questions, it’s important to have a thorough understanding of the principles of criminal law and procedure, the definitions of various crimes, and the constitutional protections available to criminal defendants.

Criminal Law & Procedure Sample Question

A police officer, without a search warrant, stops a man on the street and searches his backpack, finding stolen property inside. The man is charged with theft. His attorney seeks to exclude the stolen property from evidence on the grounds that the search was illegal.

What is the most likely outcome?

  1. The evidence will be admitted because the stolen property is proof of the crime.
  2. The evidence will be admitted because the police officer had a good-faith belief that the man had committed a crime.
  3. The evidence will be excluded because the police officer didn’t have a warrant to search the backpack.
  4. The evidence will be excluded because the man did not give consent for the search.

The correct answer is C.

The Fourth Amendment to the U.S. Constitution protects against unreasonable searches and seizures. Generally, police need a warrant to conduct a search unless a recognized exception applies, such as the suspect giving consent or the search being incident to a lawful arrest. In this case, no such exception is stated, so the warrantless search of the backpack would likely be seen as a violation of the Fourth Amendment, and the evidence obtained would be excluded under the exclusionary rule.

Here’s why the other options are incorrect:

A is incorrect because, while the stolen property is indeed proof of the crime, it doesn’t overcome the Fourth Amendment’s requirement for a warrant or a recognized exception for conducting a search.

B is incorrect because the good-faith exception to the exclusionary rule generally applies when police conduct a search with a warrant that they believe to be valid but later turns out to be defective. In this case, no warrant was mentioned.

D is incorrect because, although consent is a valid exception to the warrant requirement, the question does not indicate that the man gave consent for the search.

Evidence Questions

The Evidence section of the MBE assesses your understanding of the rules governing the presentation of evidence in a courtroom setting. This section typically consists of approximately 25 questions, though this may vary slightly from exam to exam.

These questions test your understanding of the Federal Rules of Evidence, as they would apply in a courtroom setting. The purpose is to determine your ability to interpret and apply these rules, including determining what evidence is admissible and under what circumstances, as well as the role of the judge and the jury in a trial.

Here’s a closer look at the main topics covered in this section:

  1. Relevance and Reasons for Excluding Relevant Evidence: This includes the basic concepts of relevance, as well as the various rules that might lead to the exclusion of relevant evidence, such as the rule against hearsay, the rules concerning character evidence and other policies excluding relevant evidence.
  2. Hearsay and Circumstances of Its Admissibility: This covers the definition of hearsay, as well as the many exceptions to the hearsay rule, such as statements made for purposes of medical treatment or diagnosis, business records, and excited utterances.
  3. Privileges and Other Policy Exclusions: This includes rules about when a witness can refuse to testify, such as attorney-client privilege, doctor-patient privilege, or spousal privilege.
  4. Writings, Recordings, and Photographs: This covers rules concerning the admission of these types of evidence, including the original writing rule.
  5. Miscellaneous Rules, such as Authentication of Evidence, Expert Testimony, and Examination of Witnesses.

The Evidence questions often involve hypothetical scenarios in which you are asked to apply the Federal Rules of Evidence to determine the admissibility of certain pieces of evidence. For instance, you might be asked to determine whether a statement is hearsay and, if so, whether it falls under an exception to the hearsay rule.

To perform well on this section, it’s essential to have a firm grasp of the Federal Rules of Evidence, including the reasons for their existence and their practical application in a courtroom setting. You should also understand the basic concepts underlying these rules, such as relevance and prejudice.

Evidence Sample Question

In a trial for robbery, the prosecution seeks to introduce a statement the defendant made to his friend a week before the crime. The defendant said, “I’m so broke, I might just have to rob a bank.” The defense attorney objects.

How should the court rule?

  1. Overrule the objection because the statement is an admission by a party-opponent.
  2. Overrule the objection because the statement is an excited utterance.
  3. Sustain the objection because the statement is hearsay.
  4. Sustain the objection because the statement is not relevant.

The correct answer is A.

Under the Federal Rules of Evidence, an admission by a party-opponent is not considered hearsay and is generally admissible. In this case, the defendant’s statement could be seen as an admission that shows intent or motive, so it would be admissible under this rule.

Here’s why the other options are incorrect:

B is incorrect because an excited utterance is a statement relating to a startling event or condition made while the declarant was under the stress of excitement that it caused. Here, there’s no indication that the defendant was under the stress of a startling event when he made the statement.

C is incorrect because, while the statement is an out-of-court statement offered for the truth of the matter asserted (i.e., that the defendant was considering robbing a bank), it’s an admission by a party-opponent, so it’s not considered hearsay under the Federal Rules of Evidence.

D is incorrect because the statement is relevant. It could be seen as showing the defendant’s intent or motive to commit the robbery, which makes it relevant to the case.

Real Property Questions

The Real Property section of the MBE assesses your knowledge of the principles of property law and its many nuances. Approximately 25 questions from this section are typically included in the exam.

These questions primarily concern the nature of property rights, the transfer of property rights, and the various legal issues that can arise in relation to property. They require you to demonstrate an understanding of both the theoretical concepts of property law and the practical aspects of applying those concepts to real-world scenarios.

Here’s a more detailed look at the main topics that are covered in this section:

  1. Ownership of Real Property: This includes the types of estates in land (e.g., fee simple, life estate), the rights and duties associated with each type, and the methods of acquiring title (e.g., by deed, by will, by adverse possession).
  2. Rights in Land of Others: This covers the different ways in which someone can have a legal interest in another person’s land, including easements, licenses, profits, and covenants.
  3. Real Estate Contracts: This includes issues related to the formation, interpretation, and enforcement of contracts for the sale of land.
  4. Mortgages and Deeds of Trust: This covers issues related to lending money secured by a mortgage or deed of trust on real property, including the rights and duties of the borrower and lender.
  5. Landlord and Tenant: This covers issues related to the lease of property, including the rights and duties of the landlord and tenant.
  6. Zoning and Other Land Use Regulations: This includes issues related to government regulation of the use of land.

The questions often involve hypothetical scenarios that require you to apply the principles of property law to resolve a legal issue or problem. For example, you might be asked to determine the nature of a party’s interest in a piece of property, to interpret the terms of a real estate contract, or to analyze the legality of a zoning regulation.

To do well in this section, it’s essential to have a thorough understanding of the concepts and principles of property law, the ability to interpret and apply those principles to complex factual scenarios, and a solid grasp of the legal terminology used in this field.

Real Property Sample Question

A man purchases a house from a woman. The woman had been living in the house for many years and had done extensive gardening on the property. Among her plantings were several rare and valuable fruit trees. When the man moves in, he discovers that the woman has dug up and removed all of the fruit trees. He sues the woman for the value of the trees.

Which of the following is the most likely result?

  1. The man will lose because the woman had the right to remove anything she planted.
  2. The man will lose because the fruit trees were not mentioned in the contract of sale.
  3. The man will lose because the woman had a right to take the trees as they were not permanently affixed to the land.
  4. The man will win because the fruit trees, once planted and growing, were considered a part of the real property and should not have been removed by the woman.

The correct answer is D.

In property law, items that are permanently affixed to the land, such as buildings or trees, generally become part of the real property. Once something becomes part of the real property, it typically should be included in the sale unless explicitly excluded in the contract.

Here’s why the other options are incorrect:

A is incorrect because even though the woman planted the trees, once they’re planted and growing, they’re considered a part of the real property and should not have been removed when she sold the house.

B is incorrect because while specific items included or excluded in a sale should be mentioned in the contract, this doesn’t generally apply to items like trees that are considered a fixed part of the property.

C is incorrect because the trees, once planted and growing, were likely considered permanently affixed to the land and part of the real property.

Torts Questions

The Torts section of the MBE evaluates your understanding of the principles of tort law, which deals with civil wrongs that cause harm to others. This portion of the test usually comprises approximately 25 questions.

Tort law covers a wide array of topics, with questions focusing on your understanding of legal concepts, ability to analyze facts and apply relevant law, and your knowledge of exceptions and defenses. Here are some main topics:

  1. Intentional Torts: These questions test your understanding of intentional acts that cause harm to others. You might encounter questions about battery, assault, false imprisonment, trespass, conversion, and intentional infliction of emotional distress.
  2. Negligence: A significant part of tort law, negligence deals with carelessness that causes harm to others. Topics covered include duty of care, breach of duty, causation, damages, and defenses to negligence.
  3. Strict Liability and Product Liability: These questions cover circumstances where a party is held liable irrespective of their intent or negligence. This includes cases involving dangerous activities, defective products, or animals.
  4. Defamation and Privacy Torts: These questions deal with harm caused to a person’s reputation or privacy. You may be asked to assess situations involving libel, slander, intrusion, false light, and appropriation.
  5. Economic Torts: These are questions involving harm to a person’s economic interests, including torts such as interference with contractual relations.
  6. Defenses to Torts: This includes a range of defenses that can be used in response to a tort claim. Questions may cover consent, self-defense, defense of others, defense of property, necessity, assumption of risk, comparative negligence, contributory negligence, and statutory limitations.

The torts questions will often present you with a scenario in which a wrong has been committed, and you’ll need to identify the tort, understand who the liable parties are, what defenses may be applicable, and what damages may be recovered.

Doing well on this section requires a firm grasp of tort law concepts and principles, the ability to apply these to different factual scenarios, and understanding the defenses available and when they can be used.

Torts Sample Question

A man and his young daughter are visiting a zoo. The zoo has a petting section where children can feed small farm animals. The man buys a bag of feed from the zoo’s kiosk, and his daughter begins to feed the animals. Suddenly, one of the goats becomes aggressive, butting the girl with its head and causing her to fall and break her arm. The man sues the zoo on his daughter’s behalf.

Which of the following is the most likely outcome?

  1. The zoo will not be held liable because petting the animals was an inherently dangerous activity.
  2. The zoo will not be held liable because the man assumed the risk of harm to his daughter by allowing her to feed the animals.
  3. The zoo will be held liable if it can be proven that it was negligent in failing to supervise the petting section adequately.
  4. The zoo will be held liable because it is strictly liable for any harm caused by its animals.

The correct answer is C.

The zoo has a duty of care to its patrons, especially in the petting section designed for children. If the zoo failed to provide sufficient supervision to prevent such accidents, it could be found negligent, making option C the most likely outcome.

Here’s why the other options are incorrect:

A is incorrect because petting farm animals, especially in a controlled zoo setting, is not considered an inherently dangerous activity. Therefore, the zoo cannot automatically escape liability on these grounds.

B is incorrect because while assumption of risk can be a defense in some tort cases, it typically applies to activities where there is an inherent risk that is obvious or expressly stated. In this case, feeding small farm animals at a zoo, especially in a designated petting area, would not generally be considered an activity with an assumed risk of harm.

D is incorrect because while zoos may be strictly liable for harm caused by wild or dangerous animals, this rule doesn’t typically apply to domesticated animals like goats. Therefore, the zoo would not necessarily be strictly liable for the harm caused to the girl. However, if the zoo knew the particular goat had aggressive tendencies and did nothing, a different rule might apply. This option is less likely than C because it assumes a fact not given in the scenario.

MBE Preparation Strategies

Preparing for the Multistate Bar Exam (MBE) is a significant undertaking that requires a strategic approach and disciplined commitment. It’s an experience that will push you to master complex legal principles, think critically under pressure, and refine your problem-solving skills. Your performance on the MBE can profoundly impact your legal career, so it’s essential to tackle your preparation with seriousness and focus.

  1. Understand the Exam: Start your preparation journey by comprehending the structure of the exam, the subjects covered, the format of the questions, and the skills tested. A strong foundational understanding of the MBE is a cornerstone to crafting a successful study plan.
  2. Create a Study Schedule: Your time is your greatest asset when it comes to MBE preparation. A well-thought-out study schedule, based on your availability and learning style, can make a significant difference. Allocate time to each subject based on its weight on the exam and your comfort level with the topic.
  3. Focus on the Core Subjects: Spend the bulk of your study time on the seven main subjects tested on the MBE. Don’t spread yourself too thin trying to cover every conceivable legal topic.
  4. Practice, Practice, Practice: MBE questions are designed to test not just your knowledge, but also your ability to apply that knowledge in a pressured environment. Regularly solving practice questions will familiarize you with the exam’s format and help you understand how the principles of law are tested.
  5. Understand Your Mistakes: Every mistake is a learning opportunity. When you miss a practice question, take the time to understand why your answer was wrong and why the correct answer was right. This approach will improve your legal reasoning skills and prevent you from repeating the same mistakes on the actual exam.
  6. Memorization Matters: Though the MBE is an exam that tests your understanding and application of legal principles, it also requires a fair amount of memorization. Regular review of key legal rules, principles, and cases is essential.
  7. Stay Physically and Mentally Healthy: Regular breaks, physical exercise, a healthy diet, and adequate sleep will help maintain your focus and energy levels during your study period. The MBE is a marathon, not a sprint – it’s important to keep your stress levels in check.
  8. Consider Using Resources: While I won’t mention any by name, a variety of resources, including books, online platforms, and tutoring services, can provide structured guidance, practice questions, and tips for MBE preparation. Evaluate these resources carefully and choose those that align best with your learning style and budget.
  9. Take Timed Practice Exams: Practicing under conditions that simulate the actual exam can help improve your time management skills, build your endurance, and decrease anxiety on test day.
  10. Review and Adjust Your Study Plan: Keep track of your progress and regularly review your study plan. Be flexible and willing to adjust your strategies based on what is working well and what isn’t. It’s not about the hours you put in, but the output you get from those hours.

Preparation for the MBE is a highly individual journey. What works best for one person may not work as well for another. The key is to create a strategy that works for you and to stick with it. Stay disciplined, be patient, and maintain a positive mindset.

“The MBE was tough, but I felt like I was prepared. I had been scoring in the 70% range on my practice questions, and I felt like the actual MBE was similar in difficulty. I was definitely tired by the end of the day, but I felt like I had done my best.”

Source: reddit

Test Features

Purpose of the Multistate Bar Exam (MBE)

The primary purpose of the MBE is to assess the extent to which an examinee can apply fundamental legal principles and legal reasoning to analyze given fact patterns. It is a pivotal component of the bar admissions process and is designed to measure the skills you’re most likely to need in your early years of practice.

Format and Structure

The MBE consists of 200 multiple-choice questions divided into two sections of 100 questions each. You have three hours to complete each section. The exam is designed in such a way that your understanding and application of legal principles are more critical than mere memorization.

Subject Areas

The MBE covers seven key areas of law: Civil Procedure, Constitutional Law, Contracts, Criminal Law and Procedure, Evidence, Real Property, and Torts. Each subject area is tested separately, and you need to understand the underlying principles and details of each area.

Timing

The MBE is administered twice a year, typically on the last Wednesday of February and July. This structure means that you have two opportunities each year to sit for the exam.

Scoring

MBE scores are scaled, meaning your raw score (the number of questions you answered correctly) is converted to a scaled score that adjusts for potential differences in question difficulty across different exam administrations.

Validity and Recognition

The MBE is universally recognized across the United States, with most jurisdictions including it as a part of their bar examination. The score you achieve on the MBE can be critical in determining whether you pass the bar exam and are eligible for admission to the bar in your jurisdiction. Therefore, performing well on the MBE is a significant step towards a successful legal career.

Common names for the MBE Bar Exam

The Multistate Bar Exam (MBE) is commonly known by several names, such as:

  • MBE
  • Multistate
  • Bar Exam
  • Bar Test
  • The Multistate
  • Multi-state Bar Exam
  • Multistate Bar Test
  • Uniform Bar Exam (in reference to the fact that the MBE is a part of the Uniform Bar Exam, or UBE)

“I think one thing I did differently that helped the second time around was I would do 100 questions of a single MBE subject in a row, and I did that for each MBE subject. The goal is to do the questions (you can do sets of 10 or however many you have time for) using the information from your outline while using the answers to the questions to add to your outline so that you become better at that particular MBE subject. It doesn’t really matter how many of these questions you get right or wrong, what matters is you take the knowledge from the answer choices and make sure they are in your outline.”

Source: bp000000/reddit

Technical Facts

Test Fast Facts (tl;dr)

  • Multiple-choice exam with 200 questions.
  • Conducted over a single day in two 3-hour sessions.
  • Covers 7 major law areas.
  • 1 point per question, maximum raw score of 175.
  • Scores are scaled for difficulty.
  • Given twice a year, in February and July.
  • Held in U.S. testing centers, excluding Louisiana and Puerto Rico.
  • Administered in English.
  • Disability accommodations available.
  • Part of the Uniform Bar Examination (UBE).
Exam Format

The MBE is a multiple-choice examination. It consists of 200 questions, of which 175 are scored, and 25 are pretest questions that are not scored.

Duration

The test is conducted over one full day and is divided into two 3-hour sessions. Each session includes 100 questions.

Subject Areas

The MBE covers seven major areas of law: Civil Procedure, Constitutional Law, Contracts, Criminal Law and Procedure, Evidence, Real Property, and Torts.

Scoring

Each question on the MBE is worth one point, regardless of its difficulty level. The maximum raw score is 175 points, which is then converted to a scaled score. The scaling process adjusts scores to account for possible variations in difficulty across different exam administrations.

Missouri Bar Exam Exam Statistics, Average Scaled Score: 140.3. Source: howsyourdaybin/reddit
Administration

The MBE is administered over two sessions in one day. It is typically given on the last Wednesday in February and July.

Location

The MBE is conducted in specific testing centers across all jurisdictions in the United States, except for Louisiana and Puerto Rico.

Exam Language

The MBE is administered in English.

Accessibility

The MBE is designed to be accessible to all test-takers. Accommodations for examinees with disabilities are available and can be requested through the jurisdiction’s board of bar examiners.

MBE and UBE

The MBE is a component of the Uniform Bar Examination (UBE). Scores from the MBE are portable across jurisdictions that adopt the UBE.

“Once, I completed all of the MBE subjects, I took a practice test to set a base score so that I had a metric to compare future practice tests to in order to see if I was improving.

Following the practice test, I would do sets of 25 or 50 questions mixed sets during the week at night AND REVIEW. You have to review the questions you miss and keep adding to your outline. Additionally, every other weekend I would try to take a timed 100 question practice test.”

Source: bp000000/reddit

Results Scale and Interpretations

Understanding your Multistate Bar Exam (MBE) score is crucial, as it provides valuable feedback on your understanding and application of substantive legal principles. Here’s a detailed guide to help you understand the different components of your MBE score report and how it can be interpreted:

Score ComponentDefinitionImpact
Raw ScoreNumber of correctly answered questions out of 175Foundation of your score; used to calculate scaled score
Scaled ScoreScore adjusted for exam difficulty; ranges from 40 to 200The score most jurisdictions use for pass/fail decisions
Percentile RankYour performance relative to other examineesHelps you understand your standing among test-takers
Sub-ScoresEstimated scores in each subject areaHelps identify strengths and weaknesses
Score Range and Passing ScoreWhere your score lies in the overall range; passing score varies by jurisdictionCrucial for understanding your result and planning next steps
Candidate Ability AssessmentInterpretation of your legal proficiencyInfluences bar admission and can impact employment prospects
Components of the MBE Score Report. Source: iPREP
Raw Score

Your raw score is the foundation of your MBE score report. This figure reflects the actual number of questions you answered correctly out of the 175 scored questions on the MBE. The MBE comprises a total of 200 questions; however, 25 of these are unscored pretest questions used for potential inclusion in future exams. Therefore, the maximum raw score achievable is 175.

Scaled Score

To ensure fairness and comparability across different exam administrations, your raw score is converted into a scaled score. This is because the difficulty of the MBE can fluctuate from one administration to another. The scaling process statistically adjusts raw scores to a consistent scale using a method known as equating, which accounts for variations in difficulty. The mean scaled score and standard deviation might vary depending on the specific administration, but the scores typically range from 40 to 200. The scaled score is the most critical score because it is what jurisdictions typically use to determine whether a candidate has passed.

Percentile Rank

The percentile rank is a measure of your performance relative to all other examinees. If your percentile rank is, say, 85, this means you outperformed 85% of your fellow examinees who took the MBE in your administration. This gives you a clearer perspective on where you stand compared to others.

Sub-Scores

While the National Conference of Bar Examiners (NCBE) doesn’t officially provide sub-scores for the individual subject areas on the MBE, you can still estimate them by tallying the number of questions you answered correctly in each subject area. This can provide invaluable feedback about your strengths and weaknesses in the different legal topics tested on the MBE.

Score Range and Passing Score

The NCBE does not specify a “passing” score for the MBE, as the minimum passing scaled score is set by each jurisdiction and can vary. However, many jurisdictions set a passing scaled score of around 133 or higher. Understanding where your scaled score lies within this range can be instrumental in planning your next steps, whether you’re celebrating a successful result or preparing for a retake.

Assessing Candidate Ability

Your MBE score, particularly the scaled score, serves as a standardized measure of your legal competence – specifically, your ability to apply fundamental legal principles and reasoning. A higher score not only indicates a deeper understanding and stronger application of these principles, but it also can impact your bar admission chances, given the weight the MBE carries in the bar exam.

Score Utilization

In addition to determining bar eligibility, MBE scores may also be used by potential employers or others interested in assessing an examinee’s legal acumen. A high MBE score can signal to law firms and other potential employers that the candidate has strong legal reasoning skills and a comprehensive understanding of fundamental legal principles.

In conclusion, your MBE score provides a multifaceted reflection of your legal proficiency. By thoroughly understanding the different components of your score and what they signify, you can better assess your performance, strategize your study plan, and optimize your preparation for future legal career opportunities.

Experimental Questions

Of the 200 questions on the Multistate Bar Exam (MBE), 25 are classified as experimental questions. These questions are not scored and do not contribute to your final MBE score. The purpose of these experimental questions is to test out new items for future examinations.

These questions are indistinguishable from the rest of the test, so you should treat every question as if it counts towards your score. Don’t try to identify and skip these questions; your time is better spent focusing on answering each question to the best of your ability. It’s essential to remain diligent and focused throughout the exam, as you won’t know which questions are experimental and which are scored.

MBE Bar Exam Score card. Source: u/Christopher_2227/reddit

iPREP: Concise. Focused. What you need.

1
Sign up

Immediate access

2
Practice

Online self-paced

3
Pass

Ace that Test!

MBE Exam FAQs

What is the Multistate Bar Exam (MBE)?

The MBE is a full-day, standardized, multiple-choice examination developed by the National Conference of Bar Examiners (NCBE) to assess your ability to apply fundamental legal principles and reasoning.

How many questions are on the MBE and how are they divided?

The MBE consists of 200 multiple-choice questions: 175 of these are scored, and 25 are unscored pretest questions. The questions are divided into two sections of 100 questions each, covering seven key areas of law.

What subjects are tested on the MBE?

The MBE covers seven key areas of law: Civil Procedure, Constitutional Law, Contracts, Criminal Law and Procedure, Evidence, Real Property, and Torts. Each area is represented in the exam equally.

How long does the MBE take?

The MBE is a full-day test. Each of the two 100-question sections is three hours long, for a total testing time of six hours.

How is the MBE scored?

Your raw score (the number of questions you answer correctly) is converted to a scaled score to account for differences in difficulty across different exam administrations. This scaled score is used by most jurisdictions to make pass/fail decisions.

What is a passing score on the MBE?

Each jurisdiction sets its own passing score, so it can vary. However, many jurisdictions set a passing scaled score around 133 or higher.

How can I prepare for the MBE?

There are various ways to prepare, including understanding the test structure, focusing on key legal concepts, doing lots of practice questions, and following a consistent study plan.

Can I use my MBE score to waive into another jurisdiction?

Some jurisdictions may accept your MBE score through a process called Score Transfer, allowing you to waive into their jurisdiction without retaking the exam. Check with the specific jurisdiction for their rules.

Do all jurisdictions require the MBE?

Almost all U.S. jurisdictions use the MBE as part of their bar examination. Louisiana and Puerto Rico are exceptions; they do not use the MBE.

How does my MBE score affect my employment prospects?

While MBE scores are primarily used for bar admission, potential employers may view a high MBE score as an indication of strong legal reasoning skills and a comprehensive understanding of fundamental legal principles.

MBE Bar Test Tips

Here are seven practical and effective tips to help you ace the Multistate Bar Exam (MBE):

  1. Arrive Early: There’s nothing more nerve-wracking than running late on the day of your exam. Plan your route in advance and aim to arrive at the test center at least 30 minutes before the scheduled start time. This will allow you to get settled and mentally prepare for the day ahead.
  2. Fuel Your Body and Mind: Have a healthy breakfast on the day of the exam. The MBE is a long, demanding test and you’ll need all the energy you can get. Don’t forget to pack a light lunch and some snacks for the break too.
  3. Dress Comfortably: Wear comfortable and layered clothing to the exam. Test centers can be unpredictable when it comes to temperature, so dressing in layers allows you to adjust to your comfort level.
  4. Read Each Question Carefully: Misreading a question can lead to an incorrect answer. Spend enough time to thoroughly understand what the question is asking before considering the answer choices.
  5. Trust Your First Instinct: Research suggests that your first response is often the correct one. Unless you have a compelling reason to change an answer, trust your intuition and stick with your initial choice.
  6. Manage Your Time Wisely: You’ll have an average of 1.8 minutes to answer each question. Keep an eye on the clock and pace yourself to ensure you have enough time to answer all questions. If a question seems too difficult, don’t spend too much time on it. Instead, make your best guess, mark it for review, and move on.
  7. Stay Positive and Calm: The MBE is a challenging exam, but maintaining a positive mindset can improve your performance. Take a few deep breaths if you start to feel overwhelmed and remember, every question is a new opportunity to succeed.

“My top tip for the MBE is Slow down. The MBE is testing the substantive information in the different subjects, but the questions are also testing reading comprehension. Many times the wrong answer choices will seem correct, may be factually correct, may be legally correct, but are still not the best answer choice based on what the question is asking.

It’s ok to go long on some questions because you should be able to pick up the time on easier questions. I genuinely do not think timing is much of an issue on the MBE, and there is no bonus points for finishing early. So take your time, make sure you bubble correctly, and read the question correctly.”

Source: bp000000/reddit

The MBE is a significant hurdle, but with the right preparation and test-day strategy, you can conquer it. Believe in your preparation and walk into the exam with confidence.

Administration

  • Test Location: The MBE is administered in testing centers across the United States. The exact location depends on the jurisdiction in which you are seeking admission.
  • Test Schedule: The MBE is a full-day test held twice a year, typically on the last Wednesday in February and the last Wednesday in July. The day is split into two 3-hour sessions with a lunch break in between.
  • Test Format: The MBE is a multiple-choice exam conducted on paper. It does not involve any computerized or open-ended questions.
  • Test Materials: Test-takers are typically provided with pencils at the test center. Personal items, including paper, are generally not allowed in the exam room. You will be provided with scratch paper if necessary. Check with your jurisdiction for a detailed list of what is and is not permitted.
  • Cost: The cost of the MBE is usually included in the overall bar exam fee set by each jurisdiction. This can vary widely, so check with your jurisdiction for the exact fee.
  • Retake Policy: If you do not pass the MBE, you can retake it. However, the rules for retaking the exam, including waiting periods and limits on attempts, vary by jurisdiction.

Test Provider

The Multistate Bar Exam (MBE) is developed and administered by the National Conference of Bar Examiners (NCBE), a not-for-profit organization. Established in 1931, the NCBE is dedicated to developing high-quality assessments for admission to the bar in jurisdictions across the United States. With its headquarters in Madison, Wisconsin, the NCBE serves bar admissions authorities, courts, bar associations, and law schools across the nation and beyond.

In addition to the MBE, the NCBE produces several other assessments used for bar admission, including the Multistate Essay Examination (MEE), the Multistate Performance Test (MPT), and the Multistate Professional Responsibility Examination (MPRE). Moreover, the organization provides services such as score transfers for those seeking admission in jurisdictions other than the one where they initially took the MBE. Globally, the NCBE collaborates with foreign jurisdictions and law schools to facilitate the use of their exams. In all, the NCBE’s offerings represent a comprehensive suite of tools and services aimed at ensuring a high standard in the legal profession.

Information Sources

Disclaimer – All the information and prep materials on iPREP are genuine and were created for tutoring purposes. iPREP is not affiliated with the National Conference of Bar Examiners (NCBE), which is the owner of the Multistate Bar Exam (MBE), or with any of the companies mentioned above.

Free MBE practice test: Get to know what the Multistate Bar Exam (MBE) will be like by practicing with these sample questions:

Civil Procedure Practice Questions

Civil Procedure Question 1 of 6

A defendant was served with a complaint alleging negligence and seeking damages for injuries suffered by the plaintiff. The defendant’s attorney filed a timely answer denying negligence. However, in a letter sent to the plaintiff’s attorney, the defendant’s attorney inadvertently admitted the defendant’s negligence. Realizing his mistake, the defendant’s attorney quickly sent a second letter retracting the admission.

In the court, the plaintiff’s attorney seeks to introduce the first letter as an admission by the defendant. The defendant’s attorney objects, arguing that the admission was a mistake and should not be admitted.

How should the court rule on the objection?

  1. The court should sustain the objection because the admission was retracted promptly.
  2. The court should overrule the objection because the admission was made in a letter, not in a court document.
  3. The court should overrule the objection because the admission is relevant to the negligence claim.
  4. The court should sustain the objection because the admission was not part of the defendant’s answer to the complaint.

The correct answer is C. The court should overrule the objection because the admission is relevant to the negligence claim.

  • (A) is incorrect. An inadvertent admission, once made, cannot typically be undone simply by a prompt retraction.
  • (B) is incorrect. While the letter is not a court document, it is correspondence between attorneys related to the case. Such admissions can be used in court.
  • (C) is the correct answer. The court should overrule the objection because the attorney’s admission is indeed relevant to the negligence claim and, therefore, admissible.
  • (D) is incorrect. The admission does not have to be part of the defendant’s answer to be admissible. Admissions can come from a variety of sources, including attorney correspondence related to the case.
Civil Procedure Question 2 of 6

A company incorporated and primarily based in State A manufactures a product that is sold nationwide. A consumer from State B purchased one of the company’s products online and was injured when using it. The consumer filed a negligence suit against the company in a federal court in State B. The company moved to dismiss the suit for lack of personal jurisdiction.

How should the court rule on the company’s motion?

  1. The court should grant the motion because the company is not headquartered in State B.
  2. The court should deny the motion because the company sells its products nationwide.
  3. The court should grant the motion because the product was purchased online, not in a physical store in State B.
  4. The court should deny the motion because the injury occurred in State B.

The correct answer is B. The court should deny the motion because the company sells its products nationwide.

  • (A) is incorrect. The location of the company’s headquarters is not the sole factor for determining personal jurisdiction.
  • (B) is correct. The court should deny the motion because the company intentionally avails itself of the nationwide market, which includes State B. Therefore, it is subject to personal jurisdiction in State B.
  • (C) is incorrect. The method of sale, online or in a physical store, does not automatically determine personal jurisdiction.
  • (D) is incorrect. While the injury’s location is a factor, it alone does not determine personal jurisdiction. The company’s nationwide business activity, which includes State B, is a crucial factor for establishing personal jurisdiction.
Civil Procedure Question 3 of 6

A man from State A brought a breach of contract action against a woman from State B in a State A court. The contract was formed in State B, where both parties were residing at the time. The woman moved to State A after the contract was formed. The woman wishes to remove the case to a federal court.

Can she successfully remove the case to federal court?

  1. No, because the woman now resides in the same state as the man.
  2. No, because the contract was formed in State B.
  3. Yes, because the contract was formed in State B.
  4. Yes, because diversity jurisdiction requirements are met.

The correct answer is A. No, because the woman now resides in the same state as the man.

  • (A) is correct. For diversity jurisdiction to exist and allow removal to federal court, the lawsuit must be between citizens of different states. Here, at the time of the lawsuit, both parties are residents of the same state, even though they were from different states at the time of the contract.
  • (B) is incorrect. The location where the contract was formed is not the primary factor for deciding if the case can be removed to federal court.
  • (C) is incorrect. The location where the contract was formed doesn’t govern the ability to remove the case to federal court.
  • (D) is incorrect. Although the diversity jurisdiction does require that the lawsuit is between citizens of different states and the amount in controversy exceeds $75,000, the condition of parties from different states is not met here.
Civil Procedure Question 4 of 6

An individual was injured in a car accident and filed a negligence lawsuit against the driver of the other vehicle in a federal court. The plaintiff’s attorney had failed to serve the defendant with a copy of the complaint within the time limit set by the Federal Rules of Civil Procedure. The defendant’s attorney filed a motion to dismiss the case for insufficient service of process.

Should the court grant the motion to dismiss?

  1. No, because the plaintiff’s attorney can still serve the defendant at a later time.
  2. Yes, because the plaintiff’s attorney did not follow the required procedure for service.
  3. No, because the defendant’s attorney was aware of the lawsuit.
  4. Yes, because the defendant did not receive a copy of the complaint.

The correct answer is B. Yes, because the plaintiff’s attorney did not follow the required procedure for service.

  • (A) is incorrect. While it’s true that service can generally be done at a later time, it must be done within the time limits set by the Federal Rules of Civil Procedure.
  • (B) is correct. The plaintiff’s attorney failed to serve the defendant within the time limits set by the Federal Rules of Civil Procedure. This is a valid ground to dismiss the case.
  • (C) is incorrect. Even if the defendant’s attorney was aware of the lawsuit, proper service is still required under the Federal Rules of Civil Procedure.
  • (D) is incorrect. While it’s true that the defendant did not receive a copy of the complaint, the more important issue is that the plaintiff’s attorney did not adhere to the time limit for service set by the Federal Rules of Civil Procedure.
Civil Procedure Question 5 of 6

An individual sues a multinational corporation in a federal court, alleging that the corporation’s negligence led to their personal injury. The corporation, incorporated in Delaware with its principal place of business in Texas, moves to dismiss for lack of personal jurisdiction. The incident happened in New York, where the corporation has a branch office, and the plaintiff is a resident of New York.

Should the court dismiss the case for lack of personal jurisdiction?

  1. Yes, because the corporation is not incorporated in New York.
  2. Yes, because the principal place of business is not in New York.
  3. No, because the corporation has a branch office in New York.
  4. No, because the incident occurred in New York, where the plaintiff resides.

The correct answer is D. No, because the incident occurred in New York, where the plaintiff resides.

  • (A) is incorrect. While the corporation is not incorporated in New York, that is not the only criteria for establishing personal jurisdiction.
  • (B) is incorrect. The principal place of business not being in New York does not preclude the court from asserting personal jurisdiction.
  • (C) is incorrect. Although the corporation has a branch office in New York, that fact alone would not be sufficient for the court to exercise personal jurisdiction.
  • (D) is correct. Given the corporation operates within New York, the tortious action took place in New York, and the plaintiff is a resident of New York, there is sufficient contact for the court to exercise personal jurisdiction.
Civil Procedure Question 6 of 6

A resident of State A enters into a contract with a company incorporated and principally doing business in State B. The contract contains a clause that any disputes arising from the contract shall be resolved in the courts of State B. A dispute arises, and the resident files a lawsuit in a federal court in State A. The company moves to dismiss the case, citing the contractual forum selection clause.

Should the court grant the company’s motion to dismiss?

  1. Yes, because the contractual forum selection clause designates State B as the forum for dispute resolution.
  2. No, because the plaintiff has the right to choose the forum in which to file the lawsuit.
  3. No, because forum selection clauses are unenforceable in federal courts.
  4. Yes, but only if the court finds that State B is a more convenient forum.

The correct answer is A. Yes, because the contractual forum selection clause designates State B as the forum for dispute resolution.

  • (A) is correct. Courts will generally uphold and enforce valid forum selection clauses in contracts, provided they are fundamentally fair and reasonable.
  • (B) is incorrect. Although plaintiffs typically have the prerogative to choose the forum, they can contractually agree to specific forum selection clauses that limit their choice.
  • (C) is incorrect. Federal courts will enforce forum selection clauses, as long as they are fundamentally fair and do not contravene public policy.
  • (D) is incorrect. While a court will consider the convenience of the forum in a motion to dismiss for forum non conveniens, this principle is not applicable in this case as the parties have contractually agreed to the forum.

Constitutional Law Practice Questions

Constitutional Law Question 1 of 6

The state of Z, concerned about the environmental impact of plastic waste, passes a law that requires all beverage containers sold in the state to be returnable and to have a minimum deposit charge. Two large beverage companies, incorporated and based in other states, file a lawsuit in federal court, arguing that this law violates the Dormant Commerce Clause.

Is this law likely to be held unconstitutional under the Dormant Commerce Clause?

  1. No, because the law does not discriminate against out-of-state interests.
  2. Yes, because the law imposes a burden on interstate commerce that is clearly excessive in relation to the putative local benefits.
  3. No, because the state has a legitimate interest in protecting its environment.
  4. Yes, because the law affects interstate commerce and is therefore within the exclusive power of Congress to regulate.

The correct answer is A. No, because the law does not discriminate against out-of-state interests.

Here’s why:

  • (A) is correct. A law is less likely to be held unconstitutional under the Dormant Commerce Clause if it does not discriminate against out-of-state interests and applies equally to in-state and out-of-state businesses.
  • (B) is incorrect. While the Dormant Commerce Clause prohibits states from passing legislation that imposes an excessive burden on interstate commerce in relation to the local benefits, it is not clear from the facts that the burden of this law would be clearly excessive in relation to its local environmental benefits.
  • (C) is correct but insufficient. While the state does have a legitimate interest in protecting its environment, this factor alone is not determinative in a Dormant Commerce Clause analysis.
  • (D) is incorrect. Although the regulation of interstate commerce is generally within the exclusive power of Congress, states are allowed to pass laws that affect interstate commerce as long as they do not discriminate against or excessively burden interstate commerce.
Constitutional Law Question 2 of 6

A city, under its police power, enacted a zoning ordinance to regulate land use in different areas. The ordinance designated an area for residential use only. A woman, who owned a property in the designated residential area, decided to open a home-based baking business. She then sued the city, alleging that the zoning ordinance violates her substantive due process rights under the Fourteenth Amendment.

What is the likely result?

  1. The city wins, because zoning ordinances are generally constitutional if they bear a rational relationship to a legitimate governmental interest.
  2. The city wins, because the zoning ordinance is a legitimate exercise of its police power.
  3. The woman wins, because the zoning ordinance restricts her right to use her property as she sees fit.
  4. The woman wins, because the zoning ordinance fails to take into account the needs of home-based businesses.

The correct answer is A. The city wins, because zoning ordinances are generally constitutional if they bear a rational relationship to a legitimate governmental interest.

Here’s why:

  • (A) is correct. Zoning ordinances are a common tool used by municipalities to control land use within their boundaries. They are generally upheld if they bear a rational relationship to a legitimate governmental interest, such as preserving the residential character of neighborhoods.
  • (B) is incorrect because it does not address the woman’s substantive due process claim. Simply stating that the zoning ordinance is a legitimate exercise of police power is not a sufficient response to a substantive due process challenge.
  • (C) is incorrect. While property owners have a right to use their property as they see fit, this right is not absolute and can be limited by laws that bear a rational relationship to a legitimate governmental interest.
  • (D) is incorrect. The zoning ordinance does not have to take into account the needs of every potential use of property. The question is whether it bears a rational relationship to a legitimate governmental interest.
Constitutional Law Question 3 of 6

A law was passed by the state that requires all private schools to admit a certain percentage of disadvantaged students. The state, in turn, will fund their education. A religious school that also operates as a private school sued the state, arguing that this law violated the Establishment Clause of the First Amendment.

What is the most likely result?

  1. The state wins, because the law does not endorse or advance any particular religion.
  2. The state wins, because the law has a secular purpose of improving access to education.
  3. The religious school wins, because the state law amounts to the government supporting a religious institution.
  4. The religious school wins, because the state law interferes with its religious beliefs.

The correct answer is B. The state wins, because the law has a secular purpose of improving access to education.

Here’s why:

  • (A) is incorrect. Although the law does not necessarily endorse or advance any particular religion, it indirectly funds a religious institution, which can be construed as a violation of the Establishment Clause if there is no secular purpose or primary effect.
  • (B) is correct. The Establishment Clause permits government action that has a clear secular purpose, does not primarily advance or inhibit religion, and does not excessively entangle government with religion. In this case, the law has a clear secular purpose, which is to improve access to education for disadvantaged students.
  • (C) is incorrect. Although the state law indirectly funds a religious institution, it does so for the secular purpose of improving access to education. Therefore, it doesn’t amount to government support for a religious institution in violation of the Establishment Clause.
  • (D) is incorrect. There’s no indication in the question that the state law interferes with the religious school’s beliefs.
Constitutional Law Question 4 of 6

A city passes an ordinance that all persons must show a government-issued identification card before voting in local elections. A group of citizens argues that the ordinance infringes their right to vote, as protected by the Equal Protection Clause of the Fourteenth Amendment. The city argues that the ordinance is necessary to prevent voter fraud.

What is the most likely result?

  1. The city wins because the ordinance has a legitimate purpose to prevent voter fraud.
  2. The city wins because there is no explicit right to vote in the Constitution.
  3. The citizens win because the ordinance infringes their fundamental right to vote.
  4. The citizens win because the ordinance imposes an undue burden on their right to vote.

The correct answer is D. The citizens win because the ordinance imposes an undue burden on their right to vote.

Here’s why:

  • (A) is incorrect. While preventing voter fraud is a legitimate government interest, the means used to achieve this interest must be narrowly tailored to avoid infringing on fundamental rights, like the right to vote.
  • (B) is incorrect. Although there is no explicit right to vote in the Constitution, the right to vote is a fundamental right that is protected by the Equal Protection Clause of the Fourteenth Amendment.
  • (C) is incorrect. Although the right to vote is a fundamental right, the question doesn’t provide enough information to conclude that the ordinance outright infringes this right.
  • (D) is correct. The courts could rule that the requirement to show a government-issued identification card before voting places an undue burden on the citizens’ right to vote, particularly for those who may have difficulty obtaining such an ID. This could potentially violate the Equal Protection Clause, depending on how the burden balances against the city’s interest in preventing voter fraud.
Constitutional Law Question 5 of 6

A new state law requires all schoolchildren to recite a pledge of allegiance to the state flag each morning. The law does not offer an option to abstain from the pledge. A group of parents whose religious beliefs conflict with pledging allegiance to any symbol, including the state flag, sue the state, arguing that the law violates their First Amendment right to the free exercise of religion.

What is the most likely outcome?

  1. The state wins because the law is neutral and generally applicable, thus satisfying rational basis review.
  2. The state wins because the parents’ claim does not implicate a fundamental right.
  3. The parents win because the law burdens their free exercise of religion.
  4. The parents win because the law discriminates against their religious beliefs.

The correct answer is C. The parents win because the law burdens their free exercise of religion.

Here’s why:

  • (A) is incorrect. Even if a law is neutral and generally applicable, it may still fail constitutional scrutiny if it burdens the free exercise of religion without a compelling governmental interest.
  • (B) is incorrect. The Free Exercise Clause of the First Amendment, which prohibits government interference with religious beliefs and practices, is considered a fundamental right.
  • (C) is correct. The law burdens the parents’ free exercise of religion by requiring their children to participate in an act (reciting the pledge) that goes against their religious beliefs, without offering an option to abstain. As a result, the law is subject to strict scrutiny and must serve a compelling government interest and be narrowly tailored to achieve that interest. Given the facts provided, it’s unlikely that the law would meet this high standard.
  • (D) is incorrect. While the law does burden the parents’ religious beliefs, the question does not provide information suggesting that the law discriminates against their beliefs in favor of others. Thus, it would be difficult to argue that the law is discriminatory based on the given facts.
Constitutional Law Question 6 of 6

The state legislature enacts a law prohibiting burning the state flag in protest. The law is challenged by a group of protesters who were arrested for violating it. They claim the law infringes on their First Amendment rights to free speech and symbolic expression.

What is the most likely outcome?

  1. The state wins because the flag is a symbol of state authority and respect for it must be maintained.
  2. The state wins because the act of burning the flag is not considered speech protected by the First Amendment.
  3. The protesters win because the law infringes upon their symbolic speech rights protected by the First Amendment.
  4. The protesters win because the law is a content-based restriction on speech.

The correct answer is C. The protesters win because the law infringes upon their symbolic speech rights protected by the First Amendment.

Here’s why:

  • (A) is incorrect. Although the flag is a symbol of state authority, the Supreme Court has held that the First Amendment protects desecration of the flag as a form of symbolic speech.
  • (B) is incorrect. The act of burning the flag in protest has been recognized by the Supreme Court as a form of symbolic speech protected by the First Amendment.
  • (C) is correct. The Supreme Court has ruled that flag desecration, including burning, is a form of symbolic speech protected under the First Amendment.
  • (D) is incorrect. Although the law does restrict a form of expression, it is not based on the content of the protesters’ speech. Rather, it targets the act of flag desecration regardless of the message behind it. Thus, it would be more accurate to describe it as a restriction on a form of symbolic expression, rather than a content-based restriction on speech. However, this does not change the fact that the law infringes upon the protesters’ First Amendment rights.

Contracts Practice Questions

Contracts Question 1 of 6

Alice, a successful artist, enters into a written contract with Bob’s Art Gallery. The contract stipulates that Alice will create and deliver 10 original paintings to Bob’s Art Gallery within a year. After delivering 5 paintings, Alice suddenly dies. Alice’s assistant, who has been trained by Alice and can replicate her style, offers to complete the remaining 5 paintings. Bob’s Art Gallery refuses the offer.

Can Bob’s Art Gallery successfully sue Alice’s estate for breach of contract?

  1. Yes, because Alice did not fulfill the entirety of the contract.
  2. No, because Alice’s death made the performance of the contract impossible.
  3. Yes, because Alice’s assistant offered to complete the contract, so performance was not impossible.
  4. No, because the contract was for personal services that can’t be delegated.

The correct answer is D. No, because the contract was for personal services that can’t be delegated.

Here’s why:

  • (A) is incorrect because although Alice didn’t complete the contract, her death created a condition of impossibility, which generally excuses contract performance.
  • (B) is partially correct because Alice’s death did make her personal performance impossible. However, this choice misses the key point that the contract was for personal services, which cannot be delegated, making choice (D) a better answer.
  • (C) is incorrect because the assistant’s offer to complete the contract doesn’t negate the fact that Alice’s personal services were contracted. The personal nature of the service makes the performance non-delegable.
  • (D) is correct. A contract for personal services is discharged upon the death of the person expected to perform the services. The fact that Alice’s style could be replicated by her assistant doesn’t change the personal nature of the contract.
Contracts Question 2 of 6

A reputable online marketplace for handmade crafts, CraftSpot, contracts with James, a well-known jewelry maker, for 50 handcrafted silver necklaces to be made exclusively for CraftSpot’s customers. The contract stipulates a delivery date two months from signing. After one month, the price of silver skyrockets, causing James’s costs to double. James informs CraftSpot that he cannot continue with the production of the necklaces unless the price is increased to cover the extra costs. CraftSpot refuses to adjust the price.

Can James legally cancel the contract due to the unforeseen increase in silver prices?

  1. Yes, because the contract has become commercially impracticable.
  2. No, because CraftSpot is not responsible for the change in silver prices.
  3. Yes, because CraftSpot is unjustly enriched if James continues with the contract.
  4. No, because the risk of price fluctuation in materials falls on James, the contract maker.

The correct answer is D. No, because the risk of price fluctuation in materials falls on James, the contract maker.

Here’s why:

  • (A) is incorrect. While it’s true that commercial impracticability can be a reason to void a contract, a simple increase in costs or decrease in profitability due to normal market fluctuations generally does not meet the threshold of “impracticability”.
  • (B) is true but not the best answer. While CraftSpot is not responsible for the change in silver prices, the primary issue is who bears the risk of this change under the terms of the contract.
  • (C) is incorrect. Although it might seem that CraftSpot is being enriched, they are merely benefiting from a favorable turn in market conditions, which is not unlawful.
  • (D) is correct. Unless specified otherwise, the risk of price fluctuation in materials generally falls on the contract maker. Here, the contract didn’t include a provision for changing the price in the event of an increase in materials costs, so James bears the risk.
Contracts Question 3 of 6

Jacob enters into a contract with a renowned artist, Margaret, for a custom mural on the side of his brick-and-mortar bookstore. Margaret spends three weeks creating an intricate design and sends it to Jacob for approval. Jacob reviews the design and requests several changes, which Margaret incorporates. The revised design is again sent to Jacob, who this time approves it and instructs Margaret to proceed. After Margaret has painted half the mural, Jacob changes his mind about the design and asks her to stop painting. He refuses to pay for the half-finished mural, arguing that since the work is not complete, he does not owe her anything.

Is Jacob’s refusal to pay legally justifiable?

  1. Yes, because the mural was not completed as per the contract.
  2. No, because Margaret has substantially performed her part of the contract.
  3. Yes, because Jacob has the right to change his mind about the design.
  4. No, because Jacob approved the design before Margaret started painting.

The correct answer is B. No, because Margaret has substantially performed her part of the contract.

Here’s why:

  • (A) is incorrect. Even though the mural was not completed, Margaret has substantially performed her part of the contract by creating the design, making requested changes, and painting half the mural.
  • (B) is correct. In contract law, when a party has substantially performed, they are entitled to the agreed-upon compensation, less the value of any defects. Here, Margaret has done substantial work as per the contract, and thus should be compensated.
  • (C) is incorrect. Although Jacob has the right to change his mind, he still has to pay for the work that has been done up to that point, especially since the work was done based on his approvals.
  • (D) is true but not the best answer. Although Jacob did approve the design, the primary issue here is not his approval, but rather that Margaret had substantially performed her part of the contract.
Contracts Question 4 of 6

Jake, a local fitness enthusiast, signs up for a year’s subscription to the ‘Fit and Fab’ gym. The contract contains a clause stating that refunds are not provided under any circumstances. Two months into the subscription, Jake is forced to move to another city for work, making it impossible for him to continue using the gym. Jake asks the gym for a refund for the remaining ten months, but the gym refuses, citing the ‘no refund’ clause in the contract.

Is the gym’s refusal to provide a refund legally justifiable?

  1. Yes, because Jake voluntarily signed the contract.
  2. Yes, because the ‘no refund’ clause is part of the contract.
  3. No, because Jake’s circumstances have changed.
  4. No, because the ‘no refund’ clause is unconscionable.

The correct answer is B. Yes, because the ‘no refund’ clause is part of the contract.

Here’s why:

  • (A) is correct but not the best answer. While Jake did sign the contract voluntarily, the better reasoning is that the ‘no refund’ clause specifically addresses this situation.
  • (B) is correct. Contracts are legally binding agreements, and Jake would have been aware of the ‘no refund’ clause when he signed the contract.
  • (C) is incorrect. Although Jake’s circumstances have indeed changed, this does not automatically invalidate the terms of the contract he voluntarily entered into.
  • (D) is incorrect. While some contract terms may be considered unconscionable (or unfairly one-sided), a ‘no refund’ clause in a gym membership contract does not typically meet this threshold. This would ultimately depend on the jurisdiction and specific laws, but generally, these types of clauses are enforceable.
Contracts Question 5 of 6

Fred enters into a contract with Stellar Electronics to purchase a television. The contract states that Stellar Electronics will deliver the television within two weeks. However, the television arrives four weeks later. During this period, Fred had to rent a television for an event. He is now seeking compensation from Stellar Electronics for the rental cost.

Does Fred have a legitimate claim for compensation?

  1. Yes, because Stellar Electronics breached the contract by delivering the television late.
  2. No, because Fred should have waited for the television delivery instead of renting one.
  3. Yes, because Fred incurred additional costs due to Stellar Electronics’ breach of contract.
  4. No, because the contract did not specify any compensation for late delivery.

The correct answer is A. Yes, because Stellar Electronics breached the contract by delivering the television late.

Here’s why:

  • (A) is correct. By failing to deliver the television within the stipulated two-week period, Stellar Electronics breached the contract, which could potentially entitle Fred to damages.
  • (B) is incorrect. Fred’s decision to rent a television does not negate the fact that Stellar Electronics failed to fulfill its contractual obligation.
  • (C) is partially correct but not the best answer. While it’s true that Fred incurred additional costs due to the breach, the key issue is the breach of contract by Stellar Electronics. The rental costs could potentially be recoverable as consequential damages if they were foreseeable at the time the contract was made.
  • (D) is incorrect. Even if the contract did not specifically mention compensation for late delivery, Stellar Electronics’ failure to meet the agreed delivery timeline constitutes a breach of contract, which can lead to a damages claim.
Contracts Question 6 of 6

John, a freelance web developer, enters into a contract with a company to design a website. The company makes an upfront payment, and the contract stipulates that the remaining payment will be made after the completion of the website. However, after receiving the upfront payment, John disappears and does not complete the website.

Can the company recover the upfront payment made to John?

  1. Yes, because John breached the contract by not completing the website.
  2. No, because the company should have ensured that John was trustworthy before making an upfront payment.
  3. Yes, because John failed to fulfill his contractual obligations after receiving the upfront payment.
  4. No, because the company made the upfront payment willingly and cannot demand it back.

The correct answer is A. Yes, because John breached the contract by not completing the website.

Here’s why:

  • (A) is correct. By not completing the website as stipulated in the contract, John has breached the contract. The company can, therefore, seek to recover the upfront payment made to John.
  • (B) is incorrect. The company’s judgment of John’s trustworthiness is not relevant to the breach of contract by John.
  • (C) is also correct. However, the best answer is (A) because it directly points to the breach of contract, which is the main issue here.
  • (D) is incorrect. Even though the company made the upfront payment willingly, it did so with the expectation that John would fulfill his contractual obligations. Since John did not do so, the company can seek to recover the upfront payment.

Criminal Law & Procedure Practice Questions

Criminal Law & Procedure Question 1 of 6

Detective Smith is investigating a series of robberies in a local neighborhood. Based on eyewitness reports and circumstantial evidence, she identifies Paul as the prime suspect. Without a warrant, she goes to Paul’s house, knocks on the door, and asks if she can come in and ask a few questions. Paul invites her in. While in the house, Detective Smith sees a piece of jewelry that had been reported stolen in one of the robberies. She immediately arrests Paul.

Is the arrest valid?

  1. Yes, because Detective Smith was invited into the house.
  2. No, because Detective Smith entered the house without a warrant.
  3. Yes, because Detective Smith spotted the stolen item in plain view.
  4. No, because Paul was not informed of his rights before the arrest.

The correct answer is C. Yes, because Detective Smith spotted the stolen item in plain view.

Here’s why:

  • (A) is incorrect. While Paul’s consent allowed Detective Smith to enter the house, the arrest’s validity primarily depends on the evidence she obtained while inside.
  • (B) is incorrect. A warrant was not required in this scenario because Paul consented to Detective Smith’s entry into his house.
  • (C) is correct. The “plain view” doctrine states that if law enforcement sees incriminating evidence in plain view, they can seize the evidence and use it as grounds for an arrest. Since Detective Smith saw the stolen jewelry in plain view, her arrest of Paul is valid.
  • (D) is incorrect. The requirement to inform a suspect of their rights (as per the Miranda ruling) applies primarily to custodial interrogations, not arrests. Thus, Paul’s arrest is valid even if he wasn’t informed of his rights at the time of the arrest.
Criminal Law & Procedure Question 2 of 6

Lisa was indicted for felony assault after she hit her coworker, Susan, during a heated argument at their workplace. At the trial, Lisa’s lawyer argued that Lisa had acted in self-defense, as Susan had threatened her earlier that day. The jury, however, convicted Lisa. Unsatisfied with the outcome, Lisa appeals the decision, alleging that the jury was biased against her and that the judge improperly instructed the jury on the law of self-defense.

Is Lisa’s appeal likely to be successful?

  1. Yes, if Lisa can prove that the jury was indeed biased.
  2. Yes, if the judge gave an incorrect instruction on the law of self-defense.
  3. No, if the jury’s verdict was supported by sufficient evidence.
  4. Both (A) and (B).

The correct answer is D. Both (A) and (B).

Here’s why:

  • (A) is partially correct. Bias among the jury can be grounds for a successful appeal, but Lisa would need to provide solid evidence of such bias.
  • (B) is partially correct. Incorrect jury instructions, particularly on a law that is central to the defense’s case, can be grounds for a successful appeal. But Lisa would need to demonstrate that the instruction was incorrect and that it impacted the verdict.
  • (C) is incorrect. While the presence of sufficient evidence may support the jury’s verdict, it doesn’t automatically invalidate Lisa’s claim of jury bias or improper jury instruction.
  • (D) is correct. If Lisa can substantiate either claim – that the jury was biased or that the judge gave an incorrect instruction on self-defense – her appeal may be successful. So, both (A) and (B) are correct.
Criminal Law & Procedure Question 3 of 6

During a local convenience store robbery, two suspects were apprehended. A customer who witnessed the crime was asked to identify the suspects in a police lineup. The customer correctly identified Suspect A, but mistakenly identified an innocent bystander, Bob, as Suspect B. Based on this identification, Bob was charged and convicted for the crime. Bob later appeals his conviction, alleging misidentification.

Which of the following is correct regarding Bob’s appeal?

  1. Bob’s appeal will not be successful because the eyewitness identified him in the lineup.
  2. Bob’s appeal will be successful if he can provide substantial evidence to prove his innocence.
  3. Bob’s appeal will not be successful because the eyewitness testimony constitutes sufficient evidence.
  4. Bob’s appeal will be successful if he can demonstrate that the identification procedure was flawed.

The correct answer is D. Bob’s appeal will be successful if he can demonstrate that the identification procedure was flawed.

Here’s why:

  • (A) is incorrect. The fact that an eyewitness identified Bob in a lineup does not preclude the possibility of a successful appeal, especially if the identification procedure was flawed or if Bob can provide strong evidence of his innocence.
  • (B) is partially correct. While providing substantial evidence to prove his innocence could indeed help Bob’s appeal, it is not the only way to succeed.
  • (C) is incorrect. While eyewitness testimony can constitute strong evidence, it is not infallible and can be challenged, particularly if there is reason to believe that the identification procedure was flawed or that the eyewitness may have made a mistake.
  • (D) is correct. If Bob can show that the identification procedure was flawed – for example, if he was the only person in the lineup who closely resembled the actual suspect – his appeal may be successful. A flawed identification procedure can potentially lead to an erroneous identification and, consequently, a wrongful conviction.
Criminal Law & Procedure Question 4 of 6

A police officer spots a car speeding down a residential street and initiates a traffic stop. Upon approaching the vehicle, the officer notices an unopened bottle of whiskey on the back seat. The officer asks the driver, who seems nervous but sober, if he can search the car. The driver declines the request. The officer then calls for a canine unit, which arrives 30 minutes later and performs a sniff test around the vehicle. The dog signals a positive alert, and the subsequent search reveals illegal substances in the trunk. The driver is charged with possession.

Which of the following is correct regarding the admissibility of the discovered evidence?

  1. The evidence will be admitted because the driver refused the officer’s initial request to search the vehicle.
  2. The evidence will be admitted because the canine unit signaled a positive alert.
  3. The evidence will not be admitted because the prolonged wait for the canine unit constitutes an unreasonable search.
  4. The evidence will not be admitted because the officer had no probable cause to initiate the canine sniff test.

The correct answer is C. The evidence will not be admitted because the prolonged wait for the canine unit constitutes an unreasonable search.

Here’s why:

  • (A) is incorrect. The driver’s refusal to consent to a search does not automatically justify a warrantless search or the admission of the subsequently discovered evidence.
  • (B) is incorrect. Although a canine alert can provide probable cause for a search, the wait time for the canine unit may have unreasonably prolonged the detention and made the search unconstitutional.
  • (C) is correct. The U.S. Supreme Court has held that absent reasonable suspicion of criminal activity, police cannot extend a completed traffic stop to conduct a dog sniff, as it constitutes an unreasonable search under the Fourth Amendment. The prolonged wait for the canine unit may have unreasonably extended the detention.
  • (D) is incorrect. While the officer needed probable cause or the driver’s consent to search the vehicle, a canine sniff is not considered a “search” under the Fourth Amendment and does not usually require probable cause. However, in this case, the prolonged detention for the sniff test was potentially unconstitutional.
Criminal Law & Procedure Question 5 of 6

Defendant is on trial for theft. After the prosecutor presents its case, the defendant moves for a judgment of acquittal, arguing that the state failed to present sufficient evidence. The judge denies the motion. The defendant then presents her case, calling several witnesses and presenting physical evidence. After presenting her case, the defendant again moves for a judgment of acquittal, which the judge again denies. The jury then convicts the defendant of theft.

The defendant appeals her conviction, arguing that the trial judge erred by denying her motions for a judgment of acquittal. Is the defendant’s argument likely to succeed on appeal?

  1. Yes, because the defendant preserved her claim by moving for a judgment of acquittal after the state presented its case.
  2. No, because the defendant forfeited her claim by presenting evidence after the judge denied her first motion for a judgment of acquittal.
  3. Yes, because the defendant renewed her motion for a judgment of acquittal after presenting her case.
  4. No, because the jury’s guilty verdict supersedes the judge’s denial of the motions for a judgment of acquittal.

The correct answer is B. No, because the defendant forfeited her claim by presenting evidence after the judge denied her first motion for a judgment of acquittal.

Here’s why:

  • (A) is incorrect. Although the defendant moved for a judgment of acquittal after the state presented its case, the defendant forfeited the claim by proceeding to present her own case.
  • (B) is correct. When a defendant presents her case after a denial of a motion for a judgment of acquittal, she waives her right to appeal the denial of that motion. The reasoning is that by presenting her own case, she takes the risk that the jury will evaluate the evidence differently than the judge did.
  • (C) is incorrect. Although the defendant renewed her motion for a judgment of acquittal, the fact that she presented evidence after the denial of the first motion for a judgment of acquittal forfeits her claim on appeal.
  • (D) is incorrect. The guilty verdict by the jury does not supersede the judge’s ruling on a motion for a judgment of acquittal. Rather, the issue is whether the defendant preserved her claim for appeal, which she did not by presenting her own case after the denial of the first motion for a judgment of acquittal.
Criminal Law & Procedure Question 6 of 6

Defendant is charged with armed robbery. During the trial, the prosecution calls an eyewitness to the stand. The witness testifies that she saw the defendant robbing the victim at gunpoint. The defendant’s attorney cross-examines the witness, eliciting admissions that the witness was standing 50 feet away at the time of the robbery and that she was not wearing her glasses. The jury convicts the defendant.

On appeal, the defendant argues that his conviction should be overturned because the eyewitness’s testimony was unreliable. Is the defendant likely to succeed on his appeal?

  1. Yes, because the witness admitted to standing 50 feet away and not wearing her glasses at the time of the robbery.
  2. No, because the jury was entitled to weigh the credibility of the witness.
  3. Yes, because the jury should not have been allowed to consider unreliable eyewitness testimony.
  4. No, because the witness’s admissions on cross-examination were part of the record for the jury to consider.

The correct answer is B. No, because the jury was entitled to weigh the credibility of the witness.

Here’s why:

  • (A) is incorrect. Although the witness admitted to standing 50 feet away and not wearing her glasses, these facts go to the weight of the testimony, not its admissibility. It is the role of the jury, not the appellate court, to weigh the credibility of witnesses.
  • (B) is correct. It is up to the jury, not the appellate court, to assess the credibility of witnesses, including any weaknesses revealed on cross-examination.
  • (C) is incorrect. The appellate court will not assess the reliability of an eyewitness’s testimony. Instead, the jury is tasked with evaluating the credibility of witnesses and the weight of their testimony.
  • (D) is incorrect. Although the cross-examination is part of the record for the jury to consider, the main issue here is that the credibility and reliability of the witness’s testimony is for the jury to decide, not for the appellate court.

Evidence Practice Questions

Evidence Question 1 of 6

In a negligence lawsuit, the plaintiff calls a witness who testifies that he heard the defendant say, “I didn’t see the stop sign because I was looking at my phone.” The defendant’s attorney objects to this testimony.

The court should rule that the testimony is:

  1. Inadmissible, because it is hearsay.
  2. Inadmissible, because it is a self-serving statement.
  3. Admissible, because it is a party-opponent statement.
  4. Admissible, because it is a past recollection recorded.

The correct answer is C. Admissible, because it is a party-opponent statement.

Here’s why:

  • (A) is incorrect. Although the witness’s statement about what he heard the defendant say is technically hearsay (an out-of-court statement offered for the truth of the matter asserted), it falls within the hearsay exception for admissions by a party-opponent. Thus, it is not inadmissible for being hearsay.
  • (B) is incorrect. The concept of a “self-serving statement” applies when a party tries to admit their own out-of-court statement to benefit their case. In this situation, it’s the plaintiff who is offering the defendant’s statement, not the defendant.
  • (C) is correct. The statement by the defendant is an admission by a party-opponent, which is an exception to the hearsay rule. This means it can be admitted even though it is technically hearsay.
  • (D) is incorrect. A past recollection recorded involves a witness reading from or testifying to the contents of a document that the witness prepared when the events were fresh in their memory. That is not the case here.
Evidence Question 2 of 6

In a murder trial, a witness who is a forensic scientist testifies that, based on the evidence, it is extremely likely that the defendant committed the crime. The defense attorney objects to this testimony.

The court should rule that the testimony is:

  1. Admissible, because the witness is an expert in forensic science.
  2. Inadmissible, because it is hearsay.
  3. Inadmissible, because it usurps the role of the jury.
  4. Admissible, because it is based on scientific evidence.

The correct answer is C. Inadmissible, because it usurps the role of the jury.

Here’s why:

  • (A) is incorrect. While expert witnesses can provide opinion testimony within their field of expertise, they cannot opine on the ultimate issue of guilt or innocence in a criminal trial, as that would infringe on the role of the jury.
  • (B) is incorrect. The testimony is not hearsay. Hearsay involves out-of-court statements offered for the truth of the matter asserted, which is not the case here.
  • (C) is correct. An expert witness can provide opinion testimony on matters within their field of expertise, but they cannot offer opinions on the ultimate issue of a defendant’s guilt or innocence. That determination is reserved for the jury.
  • (D) is incorrect. Although the testimony is based on scientific evidence, the expert witness is not permitted to opine on the ultimate issue of the defendant’s guilt or innocence. This goes beyond the permissible scope of expert testimony.
Evidence Question 3 of 6

In a burglary trial, the prosecution seeks to introduce surveillance footage showing a person resembling the defendant breaking into the victim’s house. The prosecution does not have any witnesses who can testify to the authenticity of the footage. The defense objects to the introduction of the footage.

Should the court admit the footage into evidence?

  1. Yes, because the jury can determine the authenticity of the footage.
  2. No, because the prosecution has not established the chain of custody.
  3. No, because the prosecution cannot confirm the identity of the person in the footage.
  4. Yes, if the prosecution can establish that the footage is a fair and accurate representation of what it purports to depict.

The correct answer is D. Yes, if the prosecution can establish that the footage is a fair and accurate representation of what it purports to depict.

Here’s why:

  • (A) is incorrect. While the jury can make determinations based on the evidence, the admissibility of the evidence itself is determined by the judge.
  • (B) is incorrect. Although establishing the chain of custody can be important for certain types of evidence, the primary concern for admissibility of video footage is whether it accurately represents what it purports to show.
  • (C) is incorrect. It’s up to the jury to decide the identity of the person in the footage, not the judge. The footage can be admitted as long as it’s fair and accurate, and its relevance to the case is determined by the jury.
  • (D) is correct. For a piece of evidence like surveillance footage to be admissible, it doesn’t necessarily need to be authenticated by a witness. As long as the prosecution can demonstrate that it’s a fair and accurate representation of what it purports to show, the judge may allow it to be admitted into evidence. Then, the jury can determine what weight to give it.
Evidence Question 4 of 6

During a trial for robbery, the prosecution wants to introduce the defendant’s confession, which was obtained while the defendant was in police custody. The defense argues that the confession was coerced and that the defendant was not read his Miranda rights. The police officer who obtained the confession testifies that he read the defendant his rights before the interrogation and that the defendant voluntarily confessed. The court holds a hearing outside the presence of the jury to determine the admissibility of the confession.

Who has the burden of proof in this hearing, and by what standard must it be met?

  1. The prosecution must prove by a preponderance of the evidence that the confession was voluntary and the Miranda warnings were given.
  2. The defense must prove by a preponderance of the evidence that the confession was involuntary and the Miranda warnings were not given.
  3. The prosecution must prove beyond a reasonable doubt that the confession was voluntary and the Miranda warnings were given.
  4. The defense must prove beyond a reasonable doubt that the confession was involuntary and the Miranda warnings were not given.

The correct answer is A. The prosecution must prove by a preponderance of the evidence that the confession was voluntary and the Miranda warnings were given.

Here’s why:

  • (A) is correct. The prosecution has the burden of proof in a suppression hearing to show by a preponderance of the evidence that the defendant’s confession was voluntary and that he was properly read his Miranda rights.
  • (B) is incorrect. The burden of proof in these hearings falls on the prosecution, not the defense.
  • (C) is incorrect. Although the “beyond a reasonable doubt” standard is used in the trial itself to determine guilt, it’s not used in suppression hearings. In these hearings, the standard is “a preponderance of the evidence.”
  • (D) is incorrect for the same reasons as (B) and (C). The defense does not carry the burden of proof, and the standard is not “beyond a reasonable doubt.”
Evidence Question 5 of 6

During a murder trial, the prosecutor wants to present the testimony of a witness who claims to have heard the defendant confess to the crime. The defense argues that this hearsay and should not be admitted. The prosecutor counters that this is an exception to the hearsay rule as a statement against interest. The defense attorney argues that because the witness and the defendant are close friends, the witness might be lying to protect the defendant.

Who decides if the hearsay exception applies, and what is the standard of proof?

  1. The jury decides by a preponderance of the evidence whether the hearsay exception applies.
  2. The judge decides beyond a reasonable doubt whether the hearsay exception applies.
  3. The jury decides beyond a reasonable doubt whether the hearsay exception applies.
  4. The judge decides by a preponderance of the evidence whether the hearsay exception applies.

The correct answer is D. The judge decides by a preponderance of the evidence whether the hearsay exception applies.

Here’s why:

  • (A) is incorrect. Although the jury is the fact-finder in a trial, it’s the judge who decides on questions of law, including whether a hearsay exception applies.
  • (B) is incorrect. The judge does decide whether the hearsay exception applies, but the standard is not “beyond a reasonable doubt.” That’s the standard for convicting a defendant of a crime, not for evidentiary decisions.
  • (C) is incorrect because, as stated earlier, it’s the judge, not the jury, who decides on questions of law.
  • (D) is correct. The judge decides on questions of law, such as whether a hearsay exception applies, and the standard for this decision is a preponderance of the evidence. This means that the judge has to decide whether it’s more likely than not that the hearsay exception applies.
Evidence Question 6 of 6

During a robbery trial, the prosecution wants to introduce a security camera footage showing the defendant, who is wearing a unique jacket that the prosecution argues ties him to the crime scene. The defense argues that the footage is blurry and that the jacket in the video could belong to anyone. They argue it would be prejudicial to allow the jury to see the footage. The judge decides to allow the footage to be shown.

On what basis can the judge make this decision?

  1. The judge can allow the footage because the probative value outweighs the prejudicial effect.
  2. The judge can allow the footage because the footage was obtained legally and is therefore admissible.
  3. The judge can allow the footage because the defendant’s right to a fair trial is not infringed as he has an opportunity to cross-examine the footage.
  4. The judge can allow the footage because it is the jury’s job to decide on the reliability and relevance of the evidence.

The correct answer is A. The judge can allow the footage because the probative value outweighs the prejudicial effect.

Here’s why:

  • (A) is correct. The judge makes a determination about whether the probative value of the evidence (its value in proving something important in the trial) outweighs its potential prejudicial effect (its potential to unfairly sway the jury against the defendant). If the judge decides that the footage’s probative value is high enough, they can allow the evidence to be shown to the jury.
  • (B) is incorrect. While it’s true that evidence must be obtained legally to be admissible, the legality of the evidence’s acquisition isn’t the issue in this scenario. The defense isn’t arguing that the footage was obtained illegally, but rather that it could unfairly influence the jury.
  • (C) is incorrect. The right to cross-examine applies to witnesses, not pieces of evidence like a video. The defense can challenge the reliability or relevance of the footage, but they can’t “cross-examine” the footage itself.
  • (D) is incorrect. While the jury does decide on the reliability and relevance of the evidence, it’s up to the judge to decide whether the evidence can be presented to the jury in the first place.

Real Property Practice Questions

Real Property Question 1 of 6

Jonah is the owner of a residential property in a quiet neighborhood. He decides to lease the property to Tony, a music teacher, who plans to use the property as his residence and a location for giving music lessons to his students. A term in the lease prohibits the property from being used for any purpose other than residential. However, Tony continues to conduct music lessons from the property. Jonah learns about this and wants to evict Tony.

What are Jonah’s rights?

  1. Jonah can terminate the lease immediately without notice because Tony breached the lease agreement.
  2. Jonah cannot evict Tony because giving music lessons can be considered a residential activity.
  3. Jonah can terminate the lease, but he must first give Tony notice and an opportunity to stop the music lessons.
  4. Jonah has no grounds for eviction because Tony’s music lessons do not interfere with the use of the property as a residence.

The correct answer is C. Jonah can terminate the lease, but he must first give Tony notice and an opportunity to stop the music lessons.

Here’s why:

  • (A) is incorrect. Jonah may not terminate the lease immediately without notice. Landlord-tenant law generally requires a landlord to give a tenant notice and an opportunity to correct a lease violation before eviction can occur.
  • (B) is incorrect. While some jurisdictions may view giving music lessons as a residential activity, the terms of the lease specifically limit the use of the property to residential purposes only. Therefore, Tony’s use of the property for music lessons can be considered a breach of the lease.
  • (C) is correct. In most jurisdictions, Jonah would need to provide Tony with notice of the lease violation and an opportunity to correct the violation before he can initiate eviction proceedings.
  • (D) is incorrect. Whether Tony’s music lessons interfere with the use of the property as a residence is irrelevant. The lease prohibits non-residential use, and Tony’s music lessons arguably fall under that category.
Real Property Question 2 of 6

Emily purchased a house from John with a warranty deed. Two years later, Emily received notice that the state government intended to build a road through her property. The government claimed they had acquired an easement by prescription over the land from the previous owner, John. Emily was never made aware of this easement when she purchased the property from John.

Can Emily successfully sue John for breach of the covenant of quiet enjoyment under the warranty deed?

  1. Yes, because the covenant of quiet enjoyment guarantees Emily the peaceful enjoyment of the property without disturbances.
  2. No, because the covenant of quiet enjoyment is not breached until Emily has been evicted from the property.
  3. Yes, because John should have disclosed the easement to Emily when he sold the property to her.
  4. No, because the government’s easement by prescription did not involve John directly, so he did not breach any covenant.

The correct answer is A. Yes, because the covenant of quiet enjoyment guarantees Emily the peaceful enjoyment of the property without disturbances.

Here’s why:

  • (A) is correct. The covenant of quiet enjoyment guarantees the grantee of a deed the right to enjoy the property without substantial interference. A government easement by prescription that was not disclosed at the time of the sale could constitute such a substantial interference, thereby breaching this covenant.
  • (B) is incorrect. A breach of the covenant of quiet enjoyment does not require actual eviction. A substantial interference with the use and enjoyment of the property can suffice.
  • (C) is incorrect. Although John should have disclosed the easement, the failure to disclose it does not in itself breach the covenant of quiet enjoyment. What matters is that the undisclosed easement constitutes a substantial interference with Emily’s use and enjoyment of the property.
  • (D) is incorrect. Even though the government’s easement by prescription might not have involved John directly, the easement’s existence and its non-disclosure at the time of the sale can constitute a breach of the covenant of quiet enjoyment under the warranty deed.
Real Property Question 3 of 6

Sharon enters into a contract to buy a vacant lot from Bill, with the intent to build a house on it. The contract includes a description of the lot’s boundaries. However, after the sale, a surveyor finds that a portion of the lot described in the contract is actually owned by Nancy, a neighbor.

Sharon decides to sue Bill for breach of the covenant of seisin. Will Sharon likely succeed?

  1. No, because the covenant of seisin does not guarantee the accuracy of property boundaries.
  2. Yes, because the covenant of seisin assures the grantee that the grantor owns the exact estate that he or she is purporting to convey.
  3. No, because the covenant of seisin is only relevant when there is a dispute over the title, not the boundaries of the property.
  4. Yes, because the covenant of seisin guarantees that the property is free from encumbrances.

The correct answer is B. Yes, because the covenant of seisin assures the grantee that the grantor owns the exact estate that he or she is purporting to convey.

Here’s why:

  • (A) is incorrect. The covenant of seisin does indeed involve a guarantee regarding property boundaries. If the grantor conveys an estate that he or she does not fully own, it can be seen as a breach of this covenant.
  • (B) is correct. The covenant of seisin assures the grantee that the grantor owns the exact estate that he or she is purporting to convey. Therefore, if a portion of the property was actually owned by someone else, as in this case, it’s a breach of this covenant.
  • (C) is incorrect. The covenant of seisin can apply in situations where there are disputes over boundaries if those disputes mean that the grantor didn’t own the full estate that he or she claimed to.
  • (D) is incorrect. The covenant of seisin does not specifically deal with encumbrances. There is a separate covenant for that, known as the covenant against encumbrances.
Real Property Question 4 of 6

Joe, a farmer, sells a parcel of his land to Mary. The land has a path that leads to a beautiful lake which Joe has been using for years to fetch water for his crops. After the sale, Mary fences the entire parcel, including the path, preventing Joe from accessing the lake. Joe sues Mary, claiming he has an easement by necessity over the path.

Will Joe likely succeed?

  1. No, because Joe did not reserve the right to the easement in the sale contract.
  2. Yes, because an easement by necessity is automatically implied when the easement is necessary for the use and enjoyment of the land.
  3. No, because Mary has the right to exclusive use of her property.
  4. Yes, because Joe had been using the path for years before he sold the land to Mary.

The correct answer is B. Yes, because an easement by necessity is automatically implied when the easement is necessary for the use and enjoyment of the land.

Here’s why:

  • (A) is incorrect. While it is generally a good practice to expressly reserve an easement in a sale contract, an easement by necessity can be implied by law even without an explicit reservation, particularly when the easement is necessary for the use and enjoyment of the land.
  • (B) is correct. An easement by necessity is automatically implied when the easement is necessary for the use and enjoyment of the land. In this case, Joe needs access to the lake to water his crops, so he has a strong argument for an easement by necessity.
  • (C) is incorrect. While it’s true that property owners generally have the right to exclusive use of their property, this right can be subject to certain limitations, such as easements by necessity.
  • (D) is incorrect. While prior use of the path by Joe might support a claim for an easement by prescription or implication, it does not automatically establish an easement by necessity. Easements by necessity are based on the needs of the land, not on the length of use.
Real Property Question 5 of 6

Anne, a real estate developer, wants to construct a shopping mall on a piece of land she owns. However, a historic house is located on the land, and local laws prohibit its demolition. To proceed with her plan, Anne sells the land to Bob, a friend, with a verbal agreement that Bob will demolish the house, then sell the land back to Anne. Bob demolishes the house, but then refuses to sell the land back to Anne.

Can Anne likely enforce the verbal agreement?

  1. No, because the agreement involves an illegal act.
  2. Yes, because Bob acted in bad faith.
  3. No, because contracts for the sale of land must be in writing to be enforceable.
  4. Yes, because Bob has unjustly enriched himself at Anne’s expense.

The correct answer is C. No, because contracts for the sale of land must be in writing to be enforceable.

Here’s why:

  • (A) is incorrect. Even though Anne’s intent to have the house demolished might violate local laws, this does not necessarily render the contract with Bob unenforceable. The legality of the underlying act is a separate issue from the enforceability of the contract itself.
  • (B) is incorrect. While Bob’s refusal to sell the land back might be viewed as bad faith, this does not overcome the requirement that contracts for the sale of land must be in writing.
  • (C) is correct. Under the statute of frauds, which is a legal principle adopted in many jurisdictions, contracts for the sale of land must be in writing to be enforceable. Because Anne’s agreement with Bob was verbal, it is likely unenforceable.
  • (D) is incorrect. While it might be argued that Bob has unjustly enriched himself, this alone does not make the verbal agreement enforceable. The requirement for a written contract for the sale of land still applies.
Real Property Question 6 of 6

Lucas, a landowner, entered into a written lease agreement with Monica for a two-year term. Lucas dies one year into the lease term. Monica stops paying rent because she believes the lease terminated upon Lucas’s death. She is sued for non-payment of rent by Lucas’s estate.

What is the likely outcome of the lawsuit?

  1. Monica will likely lose, as the lease is generally unaffected by the death of the landlord.
  2. Monica will likely win, as the lease was personal to Lucas and terminated upon his death.
  3. Monica will likely lose, as the lease is a contract, and contracts always survive the death of a party.
  4. Monica will likely win, as she is not required to pay rent to an estate.

The correct answer is A. Monica will likely lose, as the lease is generally unaffected by the death of the landlord.

Here’s why:

  • (A) is correct. In general, the death of a landlord does not terminate a lease. Unless the lease agreement specifically states that it terminates upon the landlord’s death, the lease typically survives and becomes part of the landlord’s estate. Therefore, Monica is still obligated to pay rent as per the lease agreement, and failure to do so could result in a successful lawsuit against her.
  • (B) is incorrect. A lease is not necessarily personal to the landlord. Unless explicitly stated in the lease agreement, the lease will not terminate upon the death of the landlord.
  • (C) is incorrect. While it’s true that contracts usually survive the death of a party, the wording of this option is overly broad and might be misleading in some contexts. But in the context of a lease, it’s generally true that the lease agreement will continue in effect, and Monica’s obligations under the lease do not cease because of Lucas’s death.
  • (D) is incorrect. Monica is indeed required to pay rent to Lucas’s estate unless the lease agreement specifically states otherwise. The estate steps into the shoes of the deceased landlord and assumes their rights and responsibilities, including the right to collect rent.

Torts Practice Questions

Torts Question 1 of 6

Sarah, a high school teacher, notices one of her students, Ethan, browsing a website with inappropriate content during class. She confiscates his phone and keeps it until the end of the school day. Ethan is furious and accuses Sarah of theft. If Ethan sues Sarah for conversion (a type of tort that involves wrongfully taking or keeping someone else’s personal property).

What would be the likely outcome?

  1. Sarah would likely lose the case, because she kept Ethan’s phone without his consent.
  2. Ethan would likely lose the case, because Sarah, as a teacher, has the authority to confiscate student property for disciplinary reasons.
  3. Ethan would likely win the case, because conversion requires only that the defendant exercise control over the plaintiff’s property.
  4. Sarah would likely win the case, because she returned the phone at the end of the school day, and therefore did not convert it.

Answer and Explanation:

The correct answer is (B) Ethan would likely lose the case, because Sarah, as a teacher, has the authority to confiscate student property for disciplinary reasons.

Here’s why:

  • (A) is incorrect. Even though Sarah kept Ethan’s phone without his consent, that fact alone does not make her liable for conversion. She was acting within her authority as a teacher to maintain discipline in the classroom.
  • (B) is correct. Teachers generally have the authority to confiscate student property for valid disciplinary reasons. Sarah didn’t wrongfully take Ethan’s property but instead acted in accordance with typical school disciplinary procedures. So, Ethan’s claim of conversion is likely to fail.
  • (C) is incorrect. While it’s true that conversion involves the defendant exercising control over the plaintiff’s property, not every exercise of control constitutes conversion. Conversion requires a wrongful exercise of control, and in this case, Sarah’s actions were not wrongful but within the scope of her authority.
  • (D) is incorrect. The return of the property doesn’t necessarily prevent a claim for conversion. Conversion can occur even if the defendant returns the property. However, in this case, Sarah’s actions do not constitute conversion for the reasons explained above.
Torts Question 2 of 6

Jessie, a homeowner, hires Casey, a contractor, to renovate her kitchen. During the renovation, Casey accidentally knocks over and breaks a valuable antique vase belonging to Jessie. Jessie sues Casey for negligence, claiming that he owed her a duty of care to perform the renovations without damaging her property. In response, Casey argues that he did not owe Jessie a duty of care because the vase was located in an area where he was working, and Jessie failed to remove it or inform him about its value.

Who is likely to win the case?

  1. Jessie would likely win the case, because Casey has a general duty of care to avoid causing harm while performing his work.
  2. Casey would likely win the case, because he did not have a specific duty of care to avoid damaging the vase.
  3. Jessie would likely win the case, because contractors have an absolute duty of care to avoid damaging any property in their work area.
  4. Casey would likely win the case, because Jessie had a duty to remove valuable items from the work area or inform him about their presence.

Answer and Explanation:

The correct answer is (A) Jessie would likely win the case, because Casey has a general duty of care to avoid causing harm while performing his work.

Here’s why:

  • (A) is correct. In performing his work, Casey had a general duty of care to avoid causing unnecessary harm. This includes causing damage to Jessie’s personal property, such as her vase. Whether Jessie informed him of the vase’s value is irrelevant to the existence of this general duty.
  • (B) is incorrect. Even if Casey did not know about the specific value of the vase, he still had a general duty of care to avoid damaging Jessie’s property.
  • (C) is incorrect. Although contractors do have a duty of care, it is not absolute. It is based on what is reasonable under the circumstances. If the vase was in a place where Casey could not have reasonably expected to find it, for instance, he might not be held liable for its damage.
  • (D) is incorrect. While it would have been prudent for Jessie to remove valuable items from the work area or inform Casey about them, her failure to do so does not absolve Casey of his general duty of care. However, if Jessie’s negligence contributed to the accident, it could potentially reduce the damages she could recover under the doctrine of comparative negligence, depending on the jurisdiction.
Torts Question 3 of 6

Dan is driving to work one morning when his coffee spills, causing him to take his eyes off the road. During this momentary distraction, he hits a pedestrian, Pat, who is crossing the road in a marked crosswalk. Pat sues Dan for negligence. In response, Dan argues that he did not owe a duty of care to Pat because he was only momentarily distracted, and any reasonable person could have had the same reaction in that situation.

Is Dan likely to be found liable?

  1. Dan would likely be found not liable because the coffee spill is an unforeseeable accident, and it could have happened to any reasonable person.
  2. Pat would likely not succeed because pedestrians should always be cautious when crossing the road, regardless of whether they are in a marked crosswalk.
  3. Dan would likely be found liable because drivers have a duty of care to keep their eyes on the road and avoid causing harm to pedestrians.
  4. Pat would likely not succeed because Dan was not speeding or violating any other traffic laws at the time of the accident.

Answer and Explanation:

The correct answer is (C) Dan would likely be found liable because drivers have a duty of care to keep their eyes on the road and avoid causing harm to pedestrians.

Here’s why:

  • (A) is incorrect. While the coffee spill could indeed have happened to any reasonable person, that does not absolve Dan of his duty of care. He still had a responsibility to maintain control of his vehicle and pay attention to the road, which he failed to do.
  • (B) is incorrect. While pedestrians should indeed be cautious when crossing the road, that does not relieve drivers of their duty of care to avoid hitting pedestrians. Moreover, Pat was crossing in a marked crosswalk, which gives her the right of way under most traffic laws.
  • (C) is correct. Drivers have a general duty of care to maintain control of their vehicles and avoid causing harm to others. This includes keeping their eyes on the road and watching out for pedestrians, especially in marked crosswalks. Dan’s momentary distraction does not excuse him from this duty.
  • (D) is incorrect. While Dan may not have been speeding or violating any other traffic laws, he still had a duty to avoid causing harm to pedestrians. His failure to keep his eyes on the road constitutes a breach of that duty, regardless of whether he was otherwise obeying traffic laws.
Torts Question 4 of 6

A contractor was hired to fix the roof of a house. He used a specific kind of ladder for the job, which he knew was faulty and had a high chance of collapsing under his weight. The contractor decided to use the faulty ladder anyway. While on the ladder, it broke and he fell, causing him to break his leg. The contractor then sued the homeowner, alleging that the homeowner was negligent for failing to provide a safe working environment.

Is the contractor likely to succeed in his claim?

  1. The contractor is likely to succeed because homeowners have a duty to provide a safe working environment.
  2. The contractor is not likely to succeed because he voluntarily assumed the risk by using a ladder that he knew was faulty.
  3. The contractor is likely to succeed because the homeowner should have provided him with a safer ladder.
  4. The contractor is not likely to succeed because the injury was caused by the contractor’s own negligence in using a faulty ladder.

Answer and Explanation:

The correct answer is (B) The contractor is not likely to succeed because he voluntarily assumed the risk by using a ladder that he knew was faulty.

Here’s why:

  • (A) is incorrect. Although homeowners have a general duty to provide a safe working environment, this does not extend to risks that the worker knowingly and voluntarily assumes. In this case, the contractor knew the ladder was faulty and chose to use it anyway.
  • (B) is correct. Under the doctrine of assumption of risk, a person who voluntarily and knowingly assumes a risk cannot later sue for injuries resulting from that risk. Here, the contractor knew the ladder was faulty and used it anyway, thereby assuming the risk of injury.
  • (C) is incorrect. While it would certainly have been safer for the homeowner to provide a safer ladder, the contractor’s decision to use a ladder he knew was faulty shifts the responsibility for the resulting injury to him, not the homeowner.
  • (D) is also correct. This is another way of stating the same principle as (B): the contractor’s injury was caused by his own negligence in using a faulty ladder, not by any negligence on the part of the homeowner. Therefore, his claim is unlikely to succeed.
Torts Question 5 of 6

A man went into a bakery and purchased a cake. The baker, as a gesture of goodwill, included a tiny decorative object on top of the cake. The decorative object was not edible but was commonly used in the bakery industry for decoration and was not intended to be eaten. The man, without realizing that the object was not edible, swallowed it and suffered an injury to his throat. The man then sued the baker for negligence.

Is the baker likely to be found negligent?

  1. The baker is likely to be found negligent because he did not warn the man that the object was inedible.
  2. The baker is not likely to be found negligent because the decorative object is a commonly used and accepted practice in the bakery industry.
  3. The baker is likely to be found negligent because he should have known that the man might eat the decorative object and get injured.
  4. The baker is not likely to be found negligent because the man should have exercised more caution before eating the decorative object.

Answer and Explanation:

The correct answer is (B) The baker is not likely to be found negligent because the decorative object is a commonly used and accepted practice in the bakery industry.

Here’s why:

  • (A) is incorrect. While warning the customer about the inedibility of the object would have been prudent, failure to do so does not necessarily constitute negligence. If the use of such decorative objects is a common and accepted practice in the bakery industry, the customer might be expected to know that such objects are not meant to be eaten.
  • (B) is correct. If the use of decorative, inedible objects is a commonly accepted practice in the bakery industry, then the baker may not be found negligent for using them. In such cases, the consumer is expected to exercise reasonable care and caution.
  • (C) is incorrect. For the baker to be found negligent, it must be shown that he owed a duty of care to the customer, that he breached that duty, and that the breach caused the injury. If the use of inedible decorative objects is a common and accepted practice in the industry, then the baker did not breach any duty of care.
  • (D) is also correct. This is another way of stating the same principle as (B): the customer is expected to exercise reasonable care and caution when consuming food items that may have inedible decorations. Therefore, the baker’s claim is unlikely to succeed.
Torts Question 6 of 6

Two friends, Alice and Bob, decided to have a competition to see who could stay up the longest. They both agreed that the winner would get $500 from the loser. After staying up for three consecutive days, Alice fell asleep while Bob remained awake and claimed victory. However, Alice refused to pay Bob, arguing that the competition was dangerous and therefore, the agreement was not enforceable.

Is Alice’s refusal to pay Bob justifiable under the law?

  1. Alice’s refusal to pay is justifiable because the agreement was in fact dangerous, and thus it is voidable.
  2. Alice’s refusal to pay is not justifiable because both parties voluntarily agreed to the competition, despite its dangerous nature.
  3. Alice’s refusal to pay is justifiable because the agreement is unenforceable due to the lack of consideration.
  4. Alice’s refusal to pay is not justifiable because Bob won the competition fair and square.

Answer and Explanation:

The correct answer is (A) Alice’s refusal to pay is justifiable because the agreement was in fact dangerous, and thus it is voidable.

Here’s why:

  • (A) is correct. Contracts or agreements that involve illegal or dangerous activities are generally not enforceable. Even though both parties willingly entered into the agreement, it promotes a dangerous activity, which is contrary to public policy. Therefore, the courts are likely to rule it as voidable, and Alice’s refusal to pay could be justified.
  • (B) is incorrect. Even though both parties voluntarily agreed to the competition, courts do not enforce contracts that involve illegal or dangerous activities, even if the parties agreed willingly. The law protects individuals from their bad decisions.
  • (C) is incorrect. There was consideration in this agreement – Alice’s promise to pay $500 if she lost was exchanged for Bob’s promise to pay $500 if he lost. Each party was offering something of value, so there was indeed consideration.
  • (D) is incorrect because while Bob did win the competition, the nature of the agreement itself is problematic. The law does not enforce agreements that involve dangerous or harmful activities, regardless of who won or lost.

Well done!

You have completed the Sample Questions section.

The complete iPREP course includes full test simulations with detailed explanations and study guides.

‘…TESTS THAT ACTUALLY HELP’

In the first 30 minutes of use I have learned so much more than skipping along the internet looking for free content. Don’t waste you time, pay and get tests that actually help.

Richard Rodgers

January 28, 2020 at 7:49 PM